#6 Rosh review

Pataasin ang iyong marka sa homework at exams ngayon gamit ang Quizwiz!

Question: If conservative treatment fails, what medications are considered to treat persistent symptoms in patients with meralgia paresthetica ?

Answer: Neuropathic pain medications, such as gabapentin, phenytoin or carbamazepine. Rapid Review Meralgia Paresthetica Lateral femoral cutaneous neuropathy Inguinal ligament injury Dysesthesia/numbness of proximal anterolateral thigh Rx: loose clothing, weight loss

Question: In cases of hyperglycemia, should the sodium level be corrected prior to calculating the anion gap?

Answer: No, because the chloride level is similarly diluted.

Question: True or false: Patients with fibromyalgia have no evidence of joint or muscle inflammation on physical exam?

Answer: True. Rapid Review Fibromyalgia Patient will be a woman Complaining of widespread musculoskeletal pain for > 3 months, non-restorative sleep and generalized fatigue PE will show tenderness at > 9 of 18 anatomic sites ("trigger points") Labs will be normal Diagnosis is made clinically Treatment is education, antidepressants, avoid opioids

Question: What is the most common cause of septic arthritis in patients under 50 years of age?

Answer: Gonococcus arthritis. Rapid Review Gonococcal Urethritis Patient will be complaining of purulent uretheral discharge and dysuria Labs will show gram-negative diplococci Diagnosis is made by: Gold standard - culture on Thayer-Martin media; PCR test is both sensitive and specific Most commonly caused by Neisseria gonorrhoeae Treatment is Ceftriaxone 250 mg IM AND Azithromycin 1 g PO or doxycycline 100 mg BID x 7 days

Question: How is sickle cell disease diagnosed?

Answer: Hemoglobin electrophoresis.

Question: What is the most infectious blood born pathogen?

Answer: Hepatitis B (followed by Hepatitis C, then HIV). Rapid Review Occupational Postexposure Prophylaxis (PEP) HIV PEP given with mucous membrane exposure or skin compromise Tenofovir + emtricitabine plus raltegravir HBV: Prior vaccination: PEP not needed No prior immunization: HBIG + HBV vaccine HCV: No PEP available

Question: List some common live-attenuated vaccines?

Answer: Herpes zoster, influenza, mumps/measles/rubella, rotavirus, typhoid, vaccinia (small pox), yellow fever.

Question: What electrolyte abnormality is common in pancreatitis?

Answer: Hypocalcemia. Rapid Review Acute Pancreatitis Patient will be complaining of epigastric pain radiating to the back, nausea, and vomiting PE will show ecchymosis of left flank (GreyTurner sign), umbilical ecchymosis (Cullen sign) Labs will show elevated lipase (best) and amalyse Diagnosis is made by US and Ranson's criteria Most commonly caused by gallstones > alcohol Treatment is IV fluids

Question: What viral infection is known to greatly increase the susceptibility to bacterial tracheitis?

Answer: Influenza A.

Question: How is the Measels virus spread?

Answer: It is spread through infectious droplets and is highly contagious. Rapid Review Rubeola (Measles) Spread by respiratory droplets 3 Cs: Cough, Coryza, Conjunctivitis Maculopapular rash spreads from face down Koplik spots Fever Immunize unvaccinated close contacts Supportive care, isolation

Question: Which pharmacological treatment was recently approved by the FDA for treatment of inflammatory lesions of rosacea.

Answer: Ivermectin topical cream was approved by the FDA in December 2014. Rapid Review Rosacea Patient will be complaining of acne-like rash on the forehead, cheeks and nose that gets worse with ingestion of ETOH, hot drinks and spicy foods PE will show facial flushing, telangiectasia, skin coarsening, rhinophyma (big nose) and absence of comedones Treatment is topical metronidazole

Question: Endocarditis can be reduced in patients with ventricular septal defects with which non-pharmaceutical practice?

Answer: Maintaining good oral hygiene. Rapid Review Ventricular Septal Defect (VSD) Most common CHD L → R shunt Holosystolic murmur at LLSB Smaller defects: asymptomatic 30-50% close spontaneously

Question: Infantile dilated cardiomyopathy is most commonly caused by which etiology other than idiopathic causes?

Answer: Neuromuscular disorders such as Duchenne and Becker muscular dystrophy.

Question: What is the normal respiratory rate of newborns and 1-year-olds?

Answer: Newborns 50 breaths per minute, 1-year-olds 30 breaths per minute. Rapid Review Laryngotracheitis (Croup) Patient will be a non-toxic appearing child, 6 months to 3 years old Complaining of URI symptoms with barky, seal-like cough, inspiratory stridor, low-grade fever Labs will show steeple sign on PA view Most commonly caused by Parainfluenza virus Treatment is steroids, aerosolized epinephrine

Question: Should polyneuropathy be considered in a patient with weakness and spasticity?

Answer: No, spasticity always point towards a central nervous system disorder. Rapid Review Diabetic Neuropathy Polyneuropathy Numbness, tingling "Stocking-glove" distribution Rx: TCAs, gabapentin

Question: Can a S4 be heard in normal individuals?

Answer: No. Rapid Review Acute Decompensated Heart Failure Exertional dyspnea, orthopnea, paroxysmal nocturnal dyspnea, pitting edema S3 ↑ BNP CXR: cardiomegaly, cephalization, Kerley B lines, effusions Most useful study: echo Treatment: BiPAP: ↑ oxygenation, ↓ work of breathing, ↓ preload/afterload Nitroglycerin: ↓ preload/afterload Morphine: ↓ myocardial O2 consumption, ↓ preload Furosemide: diuresis Hypotension without signs of shock: dobutamine (may worsen hypotension) Severe hypotension with signs of shock: norepinephrine (↑ systemic vascular resistance, ↑ HR, ↑ BP, ↑ myocardial O2 demand)

Question: Which ages of presentation of Legg-Calvé-Perthes has a poor prognosis?

Answer: Older ages (> 9 years). Rapid Review Legg-Calvé-Perthes disease: Male > female Ages 4 - 10 years old Unilateral avascular necrosis of the femoral head Limp after activity

Question: What bone abnormality is common in Cushing's syndrome?

Answer: Osteoporosis. Rapid Review Cushing's Syndrome: Patient will be complaining of amenorrhea, central obesity, depressive symptoms, and easy bruising PE will show purple striae, "moon face" (facial adiposity), "buffalo hump" (increased adipose tissue in the neck and upper back), and hypertension Diagnosis is made by 24 hr urine cortisol and testing ACTH levels Most commonly caused by exogenous steroids or hypercortisolism from ACTH-secreting pituitary tumor Comments: If cause is pituitary tumor then called Cushing's disease

Question: What is the primary treatment for a parathyroid adenoma?

Answer: Parathyroidectomy. Rapid Review Primary Hyperparathyroidism Causes: parathyroid adenoma > gland hyperplasia, cancer ↑ PTH, ↑ Ca, ↓ phosphorus ↑ Ca → "stones, bones, groans, psychiatric overtones" Only curative rx: parathyroidectomy

Question: What are some common causes of retained placenta?

Answer: Placenta accreta, increta and percreta; three different forms of abnormal placenta-uterus separation. Rapid Review Postpartum Hemorrhage Major cause of maternal mortality Uterine atony most common cause Immediate (> 500 cc within 24 hrs): uterine atony, retained placenta, genital trauma Rx: bimanual massage, IV pitocin, methyprostaglandin Delayed (> 24 hours up to 2 weeks): retained placental tissue, endometritis Rx: control bleeding, pitocin, antibiotics

Question: What is the other class IC antiarrhythmic medication?

Answer: Propafenone.

Question: What are the mechanisms of pulmonary bleeding?

Answer: Pulmonary hypertension, erosion into a blood vessel, and coagulopathy. Rapid Review Hemoptysis Diastolic murmur: mitral stenosis Sudden SOB, CP: PE Trauma: pulmonary contusion Immunodeficiency, immigrant: TB Hx of TB or sarcoidosis: aspergilloma Renal dysfunction: Goodpasture's syndrome or Wegener's granulomatosis (Granulomatosis with polyangiitis) Hx of tobacco use, weight loss: malignancy Massive hemoptysis: > 600 mL of blood/24 hrs Massive hemoptysis Rx: patient in bleeding side down position, mainstem bronchus intubation

Question: What is the treatment of rectal cancer?

Answer: Radical rectal resection with or without chemotherapy and radiotherapy. Rapid Review Colorectal Cancer Second leading cause of death Third most common cancer in men and women Adenocarcinoma Risk factors: age, IBD, adenomatous polyps, FAP, HNPCC Rectosigmoid > ascending > descending Left-sided cancer: tends to obstruct Right-sided cancer: tends to bleed Iron deficiency anemia Colonoscopy CEA

Question: In addition to an absent Philadelphia chromosome, how can CML be differentiated from the reactive leukocytosis seen with an infectious process?

Answer: Reactive leukocytosis will have notably lower white blood cell counts (less than 50,000) and absent splenomegaly. Rapid Review Chronic Myelogenous Leukemia (CML) Patient will be a 30 - 60 years old Complaining of most patients asymptomatic when diagnosed PE will show splenomegaly Labs will show philadelphia chromosome t(9;22) (bcr/abl) and low leukocyte alkaline phosphatase (LAP) Treatment is allogenic HSCT (curative), imatinib Comments: Phase determined by blast percentage

Question: What are the recommendations for child safety seats for children under 2 years of age?

Answer: Rear-facing child seats.

Question: What is the Obturator sign?

Answer: Right lower quadrant pain elicited during passive right hip flexion and rotation. Rapid Review Appendicitis Patient will be complaining of fever, pain that began periumbilical then moved to RLQ, nausea and anorexia PE will show Psoas sign (RLQ pain on extension of right hip), Obturator sign (RLQ pain on internal rotation of flexed right hip), Rovsing sign (right lower quadrant pain when the left lower quadrant is palpated) Diagnosis is made by ultrasound, CT Most commonly caused by fecalith Treatment is surgery

Question: What are the most common ECG findings in hypercalcemia?

Answer: Shortening of the QTc interval, PR prolongation, and QRS widening. Rapid Review Hypercalcemia Malignancy (most common inpatient cause) Primary hyperparathyroidism (most common outpatient cause) Bones, stones, groans, psychiatric overtones ↓ QT interval Initial rx: IVF

Question: When should the tDap vaccine be administered during pregnancy?

Answer: Tdap should be administered to all pregnant women at 27-36 weeks of gestation.

Question: Which joint is most commonly affected in gout?

Answer: The metatarsal-phalangeal joint of the great toe.

Question: Which criteria are used in securing a diagnosis of bacterial endocarditis?

Answer: The modified Duke criteria. Rapid Review Endocarditis Patient will be complaining of fever, rash, cough and myalgias PE will show Fever, Roth spots, Osler nodes, Murmur, Janeway lesions, Anemia, Nailbed hemorrhages, Emboli (FROM JANE) Diagnosis is made by echocardiography and Duke's criteria Most commonly caused by: IVDA: S. aureus, tricuspid Native valve: Streptococci, mitral Treatment is antibiotics Comments: GI malignancy: S. bovis

Question: How long do tissue adhesives typically remain on the wound?

Answer: Typically, 5-10 days. Rapid Review Tissue Adhesives Rapidly applied Equivalent cosmetic result Resistance to bacterial growth Avoid high tension lacerations Use petroleum products to loosen bond

Question: How long after cessation of alcohol intake do alcohol withdrawal seizures occur?

Answer: Typically, they occur between 6 and 48 hours after stopping alcohol intake but they can occur up to 5 days later. Rapid Review Ethanol Intoxication Ataxia, gait instability, slurred speech Severe intoxication: 4 Hs (Hypotension, Hypoventilation, Hypothermia, Hypoglycemia) Usually supportive care Severe case rx: glucose, thiamine

Question: What is the difference between unconjugated and conjugated bilirubin?

Answer: Unconjugated bilirubin is unable to be excreted in the bile and rises in cases of hemolysis and liver disease. Conjugated bilirubin is excreted into the bile and rises in cases of biliary obstruction. Rapid Review Acute Hepatitis HAV: fecal-oral, shellfish, alone (no carrier), asymptomatic, acute HBV: HBsAg: active infection Anti-HBs: recovered or immunized Anti-HBc IgM: early marker of infection, positive in window period Anti-HBc IgG: best marker for prior HBV HBeAg: high infectivity Anti-HBeAb: low infectivity HCV: IVDA, chronic, cirrhosis, carcinoma, carrier HDV: dependent on HBV coinfection HEV: fecal-oral (enteric) high mortality rate among pregnant (expectant) patients, epidemics, HAV and HEV are fecal-oral: "The vowels hit your bowels" Autoimmune hepatitis: young females Alcoholic hepatitis: moderate transaminase elevation, AST>ALT Supportive rx

Question: What test may be falsely positive in malaria patients?

Answer: VDRL (Venereal Disease Research Laboratory test) Rapid Review Malaria P. falciparum (deadliest), P. ovale, P. vivax, P. malariae P. ovale, P. vivax: hepatic phase Anopheles mosquito Immigrant, traveler Irregular fevers, diaphoresis P. falciparum: cerebral malaria, Blackwater fever Uncomplicated, no resistance areas rx: chloroquine Complicated, P. falciparum rx: quinidine + doxycycline

Question: True or false: surgical removal of a nail infected with onychomycosis provides a definitive cure?

Answer: False. Rapid Review Onychomycosis Patient will be complaining of thickened and discolored toenails Diagnosis is made by KOH preparation of nail scraping Treatment is oral terbinafine Comments: Serum aminotransferases should be monitored before starting treatment with terbinafine and during the treatment due to hepatotoxicity

Question: What other antiviral medications are used in the treatment of herpes zoster infection?

Answer: Famciclovir and valacyclovir.

Question: What is the most frequent complaint in patients with chronic hepatitis C infection?

Answer: Fatigue.

Question: What is the maximum age to start Rotavirus vaccine?

Answer: For rotavirus vaccine, the maximum age for the first dose is 14 weeks 6 days, and for the final dose, the maximum age is 8 months 0 days.

Question: What kind of axis deviation is described by an ECG with a positively deflected QRS complex in Lead I and a negatively deflected QRS complex in Lead II and aVF?

Answer: Left axis deviation. Rapid Review Pulmonary Hypertension Mean PA pressure: >25/>30 mm Hg at rest/exercise Exertional dyspnea RVH CXR: tapering PAs, RVH

Question: Which incomplete spinal cord syndrome has the best prognosis of full recovery?

Answer: Brown-Séquard syndrome. Rapid Review Brown-Sequard Syndrome Patient with a history of penetrating trauma PE will show ipsilateral loss of motor, position and vibration and contralateral loss of pain and temperature Most commonly caused by spinal cord hemisection

Question: What chronic hematologic condition is often associated with priapism?

Answer: Sickle cell disease. Rapid Review Priapism Low-flow: venous, emergency Low-flow rx: ice packs, phenylephrine, aspiration High-flow: arterial, semierect, painless High-flow rx: observation Aspiration performed at 2 or 11 o'clock position Priapism after cervical spine injury: C5 fracture

Question: Why is trimethoprim-sulfamethoxazole (TMP-SMX) a poor choice as a single agent to treat cellulitis?

Answer: TMP-SMX is active against most methicillin resistant Staphylococcus aureus but has poor coverage against group A streptococci; an organism often implicated in cellulitis.

Question: Which surgical emergency can present like an indirect inguinal hernia?

Answer: Testicular torsion. Rapid Review Inguinal Hernias Bimodal: < 1 and > 40 Direct: Protrudes directly through Hesselbach's triangle and medial to the inferior epigastric artery (IEA) Bulge ↓ upon reclining Indirect: ​​Most common Protrudes through internal ring, lateral to IEA ​Medial to IEA: Direct; Lateral to IEA: Indirect (MDs dont LIe) Strangulation risk: indirect > direct Nonreducible hernia: emergent surgery consultation

Question: For which group of patients would thalidomide be an inappropriate choice of medication to manage recurrent aphthous ulcers?

Answer: Women of child-bearing potential should not use thalidomide due to significant teratogenicity. Rapid Review Aphthous Ulcers Yellow centers surrounded by red halo Buccal/lip mucosa Recurrent

Which of the following describes a patient with Brown-Séquard syndrome? Ipsilateral loss of motor strength, vibratory sensation, and proprioception with contralateral loss of pain and temperature sensation below the level of injury Ipsilateral loss of pain and temperature and contralateral loss of motor strength, vibratory sensation, and proprioception Pain, loss of temperature below the level of injury, and complete loss of motor function, but retains proprioception and the ability to sense vibration and deep pressure Preservation of motor function with loss of proprioception and vibration below the level of injury

Correct Answer ( A ) Explanation: Brown-Séquard syndrome results from hemitransection of the spinal cord with unilateral damage to the corticospinal and spinothalamic tracts. This injury is usually the result of penetrating injuries or a lateral mass fracture of the cervical spine. It is also caused by spinal cord tumors, infections, and ischemia. It is rarely seen in its pure form, typically occurring with incomplete involvement of related tracts. Prognosis is excellent, with most patients recovering. Pain, temperature below the level of injury and, and complete loss of motor function, but retains proprioception and the ability to sense vibration and deep pressure describes anterior cord syndrome (C). Ipsilateral loss of pain and temperature and contralateral loss of motor strength, vibratory sensation, and proprioception is not consistent with any incomplete cord syndrome (B). Preservation of motor function with loss of proprioception and vibration below the level of injury is consistent with posterior cord syndrome (D).

A 14-year-old girl presents with her mother for pain with chewing for several days that is so severe she does not want to eat. Her mother adds that her daughter just had braces placed last week. Three small, round ulcerations with pale grey centers and surrounding erythematous halos are noticed on her buccal mucosa; they are exquisitely tender. Which of the following is the most likely diagnosis? Aphthous ulcers Herpes stomatitis Leukoplakia Necrotizing ulcerative gingivitis

Correct Answer ( A ) Explanation: This patient's acute development of painful grey ulcerations with surrounding erythema are most likely aphthous ulcers, also called "canker sores." These are usually found on the "soft" areas of the mouth; the buccal and labial mucosa are typically affected, whereas aphthous ulcers do not form on the gingiva or hard palate. Aphthous ulcers are a very common cause of oral pain, particularly in adolescents and young adults. Their specific etiology is unknown. An associated with herpesvirus 6 has been suggested; other suggested factors including psychic stress and oral trauma, such as from ill-fitting mouth guards or orthodontic appliances. In immunocompetent patients, aphthous ulcers will usually resolve without treatment in 10-14 days. However, measures to relieve pain may be needed. Options for pain relief include use of topical corticosteroids, local analgesic gels, or previously oral amlexanox paste (now discontinued). Patients with significant pain may benefit from a 1-week course of prednisone as well. In immunocompromised patients with recurrent aphthous ulcers, preventative medications, such as cimetidine or thalidomide, may be needed. HIV-infection, Behḉet disease, and inflammatory bowel disease are often related to recurrent ulcer formation. Large or persistent aphthous ulcers with no underlying explanation should arise concern for squamous cell carcinoma, and may merit an incisional biopsy. Herpes stomatitis (B) presents with prodromal oral burning, followed by an eruption of small vesicles most commonly on the gingiva or mucocutaneous junction of the lips. These will scab over within several days and usually heal without treatment in immunocompetent patients. Leukoplakia (C) is seen as a white lesion that cannot be removed from the mucosal surface with a tongue depressor. Pain is not a common presenting symptom. Necrotizing ulcerative gingivitis (D) is an acute, painful inflammation of the gingiva and may present with bleeding, halitosis, and fever. The buccal mucosa are not frequently affected, and it is usually preceded by an episode of systemic disease or significant psychic stress.

A 48-year-old woman presents to your office with a complaint of diffuse pain. She tells you that over the past few years her pain has worsened and she also experiences daily fatigue, difficulty concentrating, and headaches. Which of the following is most likely to be found on physical exam? Decreased grip strength Multiple points of tenderness to palpation Pitting edema of lower extremities Thinning hair

Correct Answer ( B ) Explanation: Fibromyalgia is a syndrome that causes chronic, widespread pain and tenderness. The etiology of fibromyalgia is unknown and the pathogenesis has not been proven, however it is believed to be related to a dysfunction in central pain processing. In addition to complaints of pain, patients often present with fatigue, cognitive difficulties, and multiple somatic and psychiatric symptoms. Fibromyalgia is a diagnosis of exclusion and patients should be evaluated for other disorders before making the diagnosis. Physical exam findings are generally within normal limits other than multiple points of tenderness to palpation, also referred to as trigger points. Laboratory test results are within normal limits. There is no cure for fibromyalgia and management involves a multi-disciplinary approach including education, lifestyle modifications and medication. Prognosis for patients with fibromyalgia varies, with better outcomes seen when patients seek help, engage in more physical exercise, pace their activities and in those that have less guarding during physical exam. Decreased grip strength (A), joint pain and swelling, and morning stiffness are seen in patients with rheumatoid arthritis. Patients with hypothyroidism have pitting edema of the lower extremities (C) and thinning hair (D) on physical exam. Other physical exam findings of hypothyroidism can include dry skin, slowed speech and movements, macroglossia, goiter and hyporeflexia.

A 33-year-old man presents with a painful, irreducible indirect inguinal hernia. You suspect strangulation. Which of the following is the most appropriate management step? Elective hernia repair Emergent herniorraphy Hernia truss Monitor for worsening symptoms

Correct Answer ( B ) Explanation: Inguinal hernias can be described as reducible, ones which can be pushed back into the abdomen, or irreducible, ones that cannot be manually reduced. Irreducible hernias are further classified as obstructed (intestinal lumen is obstructed but not ischemic), strangulated (bloody supply becomes compressed, leading to ischemia) and incarcerated (adhesions form between the hernia sac and the intestinal wall). Smaller, minimally painful, reducible hernias should be treated conservatively, with monitoring (watchful waiting) and possibly a hernia truss. Historically, elective hernia repair surgery (herniorraphy) was recommended for reducible but moderately symptomatic inguinal hernias. As of late, it may be more common to treat even these hernias with conservative care. Emergent herniorraphy is recommended for irreducible hernias, especially if signs and symptoms of incarceration, obstruction or strangulation are present. This patient's irreducible hernia requires emergent, not elective (A), surgery, and certainly not conservative measures such as monitoring (D) or a truss (C).

A 32-year-old pregnant woman complains of burning pain on her right thigh. Neurological examination reveals hyperalgesic light touch sensation in the skin of the right, proximal lateral thigh. Further examination reveals normal right leg strength, normal distal leg sensation and a 2+ ankle reflex. Which of the following neuropathies is the most likely diagnosis? Femoral neuropathy Meralgia paresthetica S1 radiculopathy Tarsal tunnel syndrome

Correct Answer ( B ) Explanation: Meralgia paresthetica is the clinical syndrome of pain or dysesthesias, or both in the anterolateral, proximal thigh, due to a compressive neuropathy of the lateral femoral cutaneous nerve. Entrapment of this nerve under the inguinal ligament is common. Mean age of onset is 50-years-old, occurring equally in men and women. It is more common in patients with diabetes mellitus, obesity (higher Body Mass Index) and pregnancy, or those who have inguinal scar tissue or wear tight belts around the waist. Diagnosis is based on the above pain description, sensory abnormalities in the nerve's distribution and absence of abnormal lower leg neurologic findings. Electrodiagnostic findings are variable, as it is difficult to perform nerve conduction studies in obese individuals. Relief of pain with local injection of anesthetic confirms the diagnosis. Spontaneous remission is common, and 90% of patients respond to conservative treatment, which includes: education on weight loss and avoiding tight belts/garments and local steroid injection. Surgery is rarely necessary. Femoral neuropathy (A) could give rise to anterolateral thigh numbness, and a gravid pelvis could compress the lumbosacral plexus and proximal femoral nerve. However, other symptoms would be present, such as numbness in the anterior thigh and medial lower leg, decreased knee reflex and quadricep weakness. S1 radiculopathy (C) presents with back and leg pain, decreased ankle reflex, weak plantar flexion and numbness in the posterior leg, lateral ankle and sole. Tarsal tunnel syndrome (D) is due to entrapment neuropathy of the posterior tibial nerve under the transverse tarsal ligament at the medial malleolus. Symptoms include aching, numbness and burning dysesthesias on the sole, distal foot and toes.

A 72-year-old man presents with lower back pain and significant weakness. Laboratory analysis is notable for a creatinine of 1.9 mg/dL, calcium of 10.7 mg/dL and total protein of 9.2 gm/dL. Which of the following might you expect to find? Hyperreflexia Lytic lesions on plain film Polycythemia Urinary retention

Correct Answer ( B ) Explanation: The patient's clinical presentation is suggestive of multiple myeloma. Multiple myeloma is on the differential of older patients with atraumatic back pain. It is caused by the proliferation of plasma cells causing a monoclonal immunoglobulin. Plasma cells rapidly proliferate in the bone marrow causing extensive skeletal destruction and osteolytic lesions on imaging studies. These lesions are often described as "punched out," particularly on the skull. The bony destruction also leads to hypercalcemia. Additionally, deposition of the immunoglobulin in the renal tubules begins to cause renal insufficiency. The elevated total protein is secondary to the circulating immunoglobulin in the blood. Hypercalcemia is associated with decreased reflexes and not hyperreflexia (A). In hypocalcemia, patients become hyperreflexic and may even develop tetany in severe cases. Polycythemia (C) is not a finding consistent with multiple myeloma. As the plasma cells infiltrate the bone marrow, patients become increasing anemic. Urinary retention (D) is not a common finding of multiple myeloma. As myeloma progresses with increased destruction of the vertebral column, patients may develop a cord compression syndrome which may include symptoms of urinary retention but this would be a late finding in the disease.

A patient's arterial blood gas is noted to have a pH 7.32, pCO2 32 mm Hg, HCO3 16 mmol/L. Which of the following is the correct interpretation of this arterial blood gas? Metabolic acidosis with respiratory alkalosis Pure metabolic acidosis Pure respiratory acidosis Respiratory acidosis with metabolic alkalosis

Correct Answer ( B ) Explanation: This patient has a pH 7.32 with a decreased serum bicarbonate and therefore has a metabolic acidosis. Physiologic compensation for a metabolic acidosis involves stimulation of the respiratory center, thereby increasing minute ventilation in an attempt to decrease the pCO2 and bring the pH back near 7.4. With normal respiratory compensation, for every 1 mmol/L decrease in HCO3, there should be a 1 mm Hg decrease in pCO2. If the measured value of pCO2 in higher than expected, there is a concomitant respiratory acidosis. Likewise, if the measured value of pCO2 is lower than expected, there is a concomitant respiratory alkalosis. The anticipated pCO2 can also be calculated using the formula pCO2 = (1.5 * [HCO3] + 8) +/- 2. In this case, with a HCO3 16 mmol/L, the expected pCO2 is 32 mm Hg (1.5 * 16 + 8). Because the calculated pCO2 and the measured pCO2 are the same, this patient has a pure metabolic acidosis. This patient does not have a metabolic acidosis with a respiratory alkalosis (A) because the calculated pCO2 and the measured pCO2 are the same. This patient does not have a pure respiratory acidosis (C) or a respiratory acidosis with a metabolic alkalosis (D). In a respiratory acidosis, the pCO2 is elevated, which is not the case with this patient.

A 34-year-old man presents to the Emergency Department with a painful erection that has lasted for approximately 3 hours. He denies any fever, chills, or dysuria. He has no significant past medical or surgical history. On physical exam, the penis is erect but no erythema, swelling, lesions, or discharge is noted. What is the most likely diagnosis? Epididymitis Peyronie disease Priapism Spermatocele

Correct Answer ( C ) Explanation: The patient is experiencing priapism. The chief complaint of a painful erection along with the lack of other findings in the patient's history and physical exam make priapism the most likely diagnosis. Priapism is a prolonged painful erection. It may be associated with urinary retention, hematologic conditions, or use of certain medications. It can result in complications such as infection and impotence. Priapism can be treated with a subcutaneous deltoid injection of terbutaline or with oral pseudoephedrine if the patient presents within 4 hours of the onset of the erection. Urologic consultation should be obtained for all patients presenting with priapism. Epididymitis (A) does not present as a painful erection, but rather with acute scrotal pain. It is most commonly associated with a recent UTI and often presents with associated symptoms of fever and dysuria. Peyronie disease (B) can cause painful erections, but is also associated with the development of palpable fibrous scar tissue plaques along the penile shaft as well as a deformity or curvature of the penis. A spermatocele (D) is an accumulation of sperm in the epididymis, usually resulting in an asymptomatic benign cyst that is found incidentally on exam.

Which of the following conditions would most likely result in right axis deviation on an ECG? Aortic valve stenosis Chronic hypertension Excess abdominal fat Pulmonary hypertension

Correct Answer ( D ) Explanation: Axis is the direction of depolarization in the heart, from the atria and spreading to the ventricles. Because the left ventricle makes up the bulk of ventricular depolarization, the resultant normal axis for the heart is toward the left ventricle. In pulmonary hypertension, the right side of the heart must work harder to overcome the increased resistance, or afterload, of the higher pressures in the pulmonary vasculature. This results in right ventricular hypertrophy; the direction of depolarization is now in the right axis. Aortic valve stenosis (A) is a condition where the aortic valve does not completely open, resulting in the left ventricle having to work harder to overcome this resistance. This often leads to left ventricular hypertrophy. This is associated with a left axis deviation because there is even more left ventricular muscle depolarizing. Excessive abdominal fat (C) can also cause left axis deviation because the heart may sit more horizontally because it is being pushed up by the excessive adipose tissue in the abdomen. Chronic hypertension (B) is associated with increased afterload, which leads to left ventricular hypertrophy and left axis deviation

A 17-year-old girl with no past medical history presents with pain and swelling to the left axilla for 3 days. She states this has never happened before. Physical examination reveals a 3 cm area of swelling and tenderness in the axilla with no warmth or erythema. The area is fluctuant. Which of the following represents the appropriate management? Amoxicillin and follow up Ciprofloxacin and follow up CT scan with contrast Incision and drainage

Correct Answer ( D ) Explanation: The patient presents with an uncomplicated, simple cutaneous abscess, which should be treated with incision and drainage (I&D). A simple cutaneous abscess is a localized collection of pus that results in a fluctuant soft tissue mass. They can occur in any part of the body but are most commonly found in the neck, axilla and perirectal areas. Most abscesses contain bacteria but up to 5% of them are sterile. This is most commonly seen in patients with parenteral drug abuse. The majority of cutaneous abscesses are caused by skin flora and Staphylococcus aureus is the most commonly cultured aerobic agent. Abscesses near mucous membranes (perioral and perirectal) tend to predominantly be caused by anaerobic bacteria. The standard treatment for cutaneous abscesses is incision and drainage. In patients with normal immune systems and minimal erythema after drainage, antibiotics are not indicated. For I&D, the incision should be made to adequate length and depth to ensure complete drainage. The cavity is typically left open with or without a wick for drainage. Treatment with antibiotics alone (A & B) is inadequate as the antibiotic will not be able to adequately penetrate the abscess cavity to eliminate the infection. CT scan with contrast (C) is not indicated in the workup of a simple cutaneous abscess.

Question: What tests may be ordered to confirm the diagnosis of multiple myeloma?

nswer: Serum and urine protein electrophoresis. Rapid Review Multiple Myeloma Single clone plasma cell malignancy Elderly CRAB: hyperCalcemia, Renal insufficiency, Anemia, lytic Bone lesions/Back pain X-ray: lytic lesions Monoclonal antibody spike PBS: Rouleaux formations SPEP: M spike UPEP: Bence-Jones

Question: Which infection is commonly found among patients with recent antibiotic use or hospitalization and causes diarrhea?

Answer: Clostridium difficile. Rapid Review Salmonellosis Patient with a history of eating poultry, meat, or eggs Complaining of fever, bloody diarrhea, and abdominal cramps Labs will show fecal WBCs Comments: common cause of osteomyelitis in children with sickle cell disease

Question: What is the effect of mixing metronidazole with alcohol?

Answer: Disulfiram-like reaction.

Question: Consumption is which type of beverage may transiently improve symptoms of benign essential tremor?

Answer: Essential tremor tends to temporarily improve after patients consume a small amount of alcohol. Rapid Review Essential Tremor Patient with a history of a family member with similar symptoms Complaining of hand tremor that is exacerbated by action and improved after alcohol consumption Most commonly caused by autosomal dominant Treatment is propanolol

Question: What percentage of children with Kawasaki disease who are not treated will go on to develop coronary artery aneurysms?

Answer: 15-25%. Rapid Review Kawasaki Disease Patient will be a child < 4 years old With a history of high fever for 5 days Complaining of conjunctivitis, rash, adenopathy, strawberry tongue, hand/feet edema, fever Treatment is IVIG + aspirin Comments: #1 cause of pediatric acquired heart disease, risk for coronary artery aneurysm Mnemonic: CRASH and burn: Conjunctivitis, Rash, Adenopathy, Strawberry tongue, Hand/feet edema, Fever

Question: What is arteriovenous nicking?

Answer: A finding of hypertensive retinopathy, AV nicking refers to the appearance of retinal arteries indenting or displacing the retinal veins, indicative of vessel wall thickening. Rapid Review Diabetic Retinopathy Patient with a history of diabetes Complaining of progressive central vision loss and of seeing "red spots" and "floaters" Fundoscopic exam will show microaneurysms, hypervascularization, flame-shaped and splinter hemorrhages, and cotton-wool spots Treatment is glucose control, laser surgery or vitrectomy

Question: What medical treatment can be tried in stage I and II uterine prolapse prior to any surgical treatment?

Answer: A pessary, usually of the ring, Smith-Hodge, donut, cube, or inflatable variety. Rapid Review Uterine Prolapse Risk factors: multiparity, age, decreasing estrogen levels, trauma Rx: Kegel exercises, pessary, surgery

Question: In patients with thrombocytopenia, what is the transfusion trigger in the absence of acute bleeding or required surgery?

Answer: A platelet level <10,000 should prompt transfusion as these patients are at increased risk of spontaneous hemorrhage. Rapid Review Heparin-Induced Thrombocytopenia (HIT) Type 1: PLT recovery with or without heparin cessation Type 2: autoimmune, venous/arterial thromboses ↓ in PLTs by 50% Stop heparin, use direct thrombin inhibitor

Question: What other lab test is required to calculate an accurate corrected calcium?

Answer: Albumin. Calcium is partially protein bound in the blood and with low albumin the proportion of free calcium is higher. Rapid Review Hypercalcemia Malignancy (most common inpatient cause) Primary hyperparathyroidism (most common outpatient cause) Bones, stones, groans, psychiatric overtones ↓ QT interval Initial rx: IVF

Question: What medication implicated in pill esophagitis carries the highest risk for subsequent stricture formation?

Answer: Alendronate.

Question: Nephrotic syndrome can also lead to the loss of protein S, protein C, & antithrombin III, predisposing the patient to what serious complication?

Answer: Arterial or venous thrombosis. Renal vein thrombosis is the most common thrombotic sequelae.

Question: At what age should a child with sickle cell anemia get additional pneumococcal vaccines with the 23-valent pneumococcal vaccine?

Answer: At 2 and 5 years of age.

Question: What changes in the brain are seen with Huntington disease?

Answer: Atrophy of the caudate nucleus and putamen.

Question: What is the epidemiology of benign prostatic hyperplasia?

Answer: Begins in 30s; 50% of men affected by age 50 years; 75% affected by 80 years (with nearly half of these cases being clinically significant). Rapid Review Benign Prostatic Hyperplasia (BPH) Patient will be older Complaining of Hesitancy, Intermittence/Incontinence, Frequency/Fullness, Urgency, Nocturia (HI FUN) PE will show a soft, smooth and mobile prostate without any nodules or indurations Diagnosis is made by digital rectal exam Most commonly caused by stromal and epithelial cell growth in the transitional zone of the prostate Treatment is α-blockers, 5α-reductase inhibitors, surgery (TURP)

Question: Name 4 other causes of hypercalcemia?

Answer: Calcium supplementation, Hyperparathyroidism, Iatrogenic, Paget's disease, Acromegaly, Addison's disease, Milk-alkali-syndrome, Zollinger-elison syndrome, Excessive vitamin A and D, Sarcoidosis.

Question: What other antibiotic can be used to treat Lyme disease in a pregnant patient?

Answer: Cefuroxime axetil, a second-generation oral cephalopsporin. Rapid Review Lyme Disease NE USA Campers/hikers Ixodes tick harbors Borrelia burgdorferi Stage I: erythema migrans (pathognomonic), viral-like syndrome Stage II: arthritis, myocarditis, bilateral Bell's palsy Stage III: chronic arthritis, chronic encephalopathy Rx: doxycycline CNS/cardiac involvement: IV ceftriaxone Pregnant/children under 8 rx: amoxicillin

You are examining an elderly man who complains of urinary frequency and hesitancy. During a digital rectal examination, palpation of the anterior rectal wall offers the best approach in evaluating which of the following structures? Prostate gland Testicle Ureter Urethral meatus

Correct Answer ( A ) Explanation: Benign prostatic hyperplasia (BPH) is also known as benign enlargement of the prostate. It is incorrectly referred to by many as benign prostatic hypertrophy, as the process is based on increased number, and not increased size, of prostatic cells. It involves stromal and epithelial cell hyperplasia which results in adenofibromatous nodules. As the nodules enlarge, and prostate specific antigen levels rise, the urethra compresses and leads to urinary tract obstruction and infections, urinary hesitancy, frequency and retention. Detection and monitoring are accomplished with a digital rectal examination. Physical examination begins with positioning the patient in a fetal position, or bent forward over a table or desk. Continue with inspection of the perineum looking for mass, erythema, edema and rashes. Palpation follows with a generally lubricated digit inserted into the anus. Rotation of the examiner's digit in the distal anorectal canal allows for assessment of mass, nodules, induration or irregularities. Palpation of the posterior, usually superior (if patient is bent over a table), surface mainly evaluates the rectal wall. Anterior (or inferior, if patient is bent over a table) rectal wall palpation not only evaluates the rectum but also the prostate, which sits anterior to the anterior rectal wall. End the examination by placing some stool on a hemoccult card to look for blood. Testicular (B) examination is accomplished with palpation of the scrotum, not the rectum. The ureter (C) connects the kidneys to the urinary bladder. Its deep location usually makes it inaccessible to an examiner's digit during rectal examination. The urethral meatus (D) is the distal opening of the urethra. It is best examined during palpation and inspection of the glans penis.

Which of the following drugs is most likely to be associated with the development of atrial tachydysrhythmias? Ethanol Gamma hydroxybutyrate (GHB) Lorazepam Phenobarbital

Correct Answer ( A ) Explanation: Ethanol abuse is associated with the development of atrial dysrhythmias, specifically, atrial fibrillation. Alcohol ingestion (acute or chronic) has multiple effects on the cardiovascular system. It can exacerbate coronary artery disease, lead to cardiomyopathy and produce dysrhythmias. Left ventricular dysfunction is common in patients with moderate alcohol consumption. Additionally, these patients may have diastolic dysfunction. Supraventricular and ventricular dysrhythmias are common. The so called "holiday heart" that occurs with heavy drinking can present as atrial fibrillation or, in unusual cases, ventricular tachycardia. Additionally, electrolyte deficiencies (hypokalemia and hypomagnesemia) predispose to dysrhythmias. GHB (B) has been associated with bradycardia and hypotension. Lorazepam (C) and phenobarbital (D) are generally not associated with cardiac dysrhythmias.

A previously healthy 30-year-old man presents to your clinic with questions about hepatitis C screening. He has no history of intravenous drug use, blood transfusions, or needlestick injury. Which of the following is the most appropriate next step in management? Reassurance and counseling on prevention Referral to gastroenterology Screening with hepatitis C antibody test Screening with hepatitis C RNA test

Correct Answer ( A ) Explanation: Hepatitis C (HCV) infection can lead to cirrhosis of the liver and end-stage liver disease. Risk factor determination and screening for HCV are included as a part of overall preventive health maintenance. Risk factors for HCV infection include history of intravenous drug use or intranasal cocaine use, blood transfusion or receiving organ donation prior to July 1992, children of mothers who have HCV, needlestick injuries or exposure to HCV-positive blood, previous HIV infection, incarceration, sexual contact with a HCV-positive partner, and patients with evidence of liver disease. Patients determined to be at-risk for HCV should be screened. A one-time screening is also recommended for patients born between 1945 and 1965. Given the lack of risk factors in this patient, screening is not recommended and you should provide reassurance and counseling on prevention. Patients found to be positive for HCV should be referred to gastroenterology (B) for follow up. Patients at high-risk for HCV or those born between 1945 and 1965 should undergo initial screening with the HCV antibody test (C). A positive HCV antibody test should be followed by the HCV RNA test (D) to determine the genotype of the virus. This helps determine treatment options.

What is the genetic basis of Huntington disease? Expansion of the cytosine-adenine-guanine trinucleotide repeats in the HTT gene Hemizygous deletion of up to 28 genes on chromosome 7q11.23 Mutations in the fibrillin gene Mutations in the gene encoding phenylalanine hydroxylase

Correct Answer ( A ) Explanation: Huntington disease (HD) is an autosomal dominant inherited, neurologic disorder characterized by choreiform movements, dementia and psychiatric disturbances. The pathophysiology of the disease is still not fully understood. Diagnosis is with genetic testing. Asymptomatic patients with a known family history of HD may also request genetic testing prior to the onset of symptoms. Genetic testing in individuals with HD will show expansion of the cytosine-adenine-guanine trinucleotide repeats in the HTT gene. There is no cure and no disease-modifying treatment available at this time. Treatment focuses on supportive care for the patient and family along with management of symptoms. HD is progressive with the end stages of the disease being severe disability and then death. Hemizygous deletion of up to 28 genes on chromosome 7q11.23 (B) is found in Williams syndrome. Clinical manifestations of Williams syndrome include cardiac anomalies, impaired cognition, friendly personality, "elfin" facies and genitourinary abnormalities. Mutations in the fibrillin gene (C) are seen in Marfan syndrome, an autosomal dominant connective tissue disorder. Mutations in the gene encoding phenylalanine hydroxylase (D) cause phenylketonuria, a disorder that if untreated causes intellectual disability.

A 57-year-old woman with multiple myeloma presents with myalgias, abdominal pain, generalized weakness, and confusion. Laboratory testing demonstrates a calcium of 15.5 mg/dL. Aggressive hydration with normal saline is initiated. Which of the following medications is also indicated? Bisphosphonate Hydrochlorothiazide Insulin Recombinant urate-oxidase

Correct Answer ( A ) Explanation: Hypercalcemia is generally a product of another underlying disorder and not a primary process in itself. Causes are grouped into four categories: malignancy (primary hematologic, metastases to the bone, or parathyroid producing tumor), hyperparathyroidism, increased intake (milk-alkali syndrome, vitamin D or A toxicity), and increased bone breakdown (immobilization, Paget disease). Clinically, patients experience lethargy, weakness, myalgias, constipation, and anorexia. The clinical presentation is often remembered by the mnemonic "bones, stones, groans and psychiatric overtones." The first step in treatment is aggressive hydration. The addition of intravenous bisphosphonates (e.g. zoledronic acid) inhibits calcium release from bone. It is usually reserved for hypercalcemia associated with malignancy. Hydrochlorothiazide (B) ​is contraindicated as it increases resorption of calcium and can lead to increased serum levels. Furosemide, a loop diuretic, is an acceptable treatment after the patient is euvolemic. Insulin (C) is used in the treatment of hyperkalemia, not hypercalcemia. Recombinant urate-oxidase (e.g. rasburicase) (D) is used in the treatment of hyperuricemia. Hyperuricemia requiring aggressive management may be seen in tumor lysis syndrome.

A 7-year-old girl presents with the rash seen above. Her mother states it appeared 7 days after hiking through the woods near their home in New Jersey. She has no known drug allergies. Which of the following is the most appropriate treatment for this patient? Amoxicillin Azithromycin Cephalexin Doxycycline

Correct Answer ( A ) Explanation: Lyme disease is the most common vectorborne disease in the United States. It is a tickborne illness caused by the spirochete Borrelia burgdorferi. The early phase of the disease results in the characteristic rash seen above, erythema migrans. Prompt treatment of early disease can shorten the duration of symptoms and prevent progression to later stages of disease. Pregnant or lactating women and children younger than 8 years of age should receive amoxicillin. Advanced or severe disease should be treated with intravenous ceftriaxone or penicillin. Azithromycin (B), a macrolide antibiotic, is not recommended as a first-line agent for early Lyme disease. It should be reserved for those who cannot tolerate doxycycline or amoxicillin. Cephalexin (C) is a first-generation cephalosporin that is ineffective against Lyme disease. Doxycycline (D) is the drug of choice for men, nonpregnant and nonlactating women, and children older than 8 years of age. Doxycycline is a tetracycline that is readily bound to calcium deposited in newly formed bone or teeth in young children. When administered during pregnancy, it can be deposited in the fetal teeth, leading to fluorescence, discoloration, and enamel dysplasia. It can also be deposited in bone, where it may cause deformity or growth inhibition.

A splinter went deep into the arm of a man while he was working in the yard. After thorough wound cleansing, you inquire about his tetanus status. He is certain that he received all of his childhood vaccines and a "tetanus booster" 4 years ago, but does not know which vaccine he received. Which one of the following is the best choice for this patient regarding tetanus immunization at this time? No immunization Tetanus immune globulin Tetanus toxoid Tetanus toxoid with reduced diphtheria

Correct Answer ( A ) Explanation: No additional vaccine is needed at this time since received his initial childhood vaccinations as well as a tetanus booster within the past 5 years. Tetanus immune globulin (B) is recommended in addition to tetanus vaccine for wounds that are tetanus-prone due to contamination and tissue damage in persons with an uncertain primary vaccine history. Tetanus toxoid (TT) (B) is usually indicated only when the diphtheria component is contraindicated, which is uncommon. The tetanus toxoid with reduced diptheria (Tdap) (C) should be administered to any person with an uncertain or incomplete history of completing a three-dose primary series of tetanus toxoid-containing vaccine, or if the most recent dose was given ≥ 10 years ago (for minor and clean wounds) or ≥ five years ago (for puncture wounds or wounds contaminated with dirt).

An 88-year-old woman presents complaining of substernal chest burning and difficulty swallowing that started earlier today. She was taking her pills when one of them "got stuck in her throat." She has an extensive medical history and is on multiple medications. Which of the following medications is most likely responsible for her symptoms? Alendronate Lisinopril Metformin Simvastatin

Correct Answer ( A ) Explanation: Pill esophagitis occurs when a pill remains in contact with esophageal mucosa for a prolonged period, leading to irritation, ulceration, or stricture formation. It is associated with many medications, especially alendronate (a bisphosphonate used in the treatment of osteoporosis), tetracyclines, NSAIDs, potassium chloride, and ferrous sulfate. Treatment includes antacids to reduce esophageal inflammation. Patients should be advised to remain in an upright position while taking their medications and to drink at least four ounces of liquid with each pill. Lisinopril (B), metformin (C), and simvastatin (D), are not commonly implicated in pill esophagitis.

You examine a four-year-old boy for a limp. The parents note that the boy was gradually refusing to run, and later, there was the presence of a limp with walking that is especially prominent at the end of the day. Parents deny fever or rash. On exam, the boy appears well with an antalgic gait and limited hip rotation. Plain radiographs obtained were normal. Bone scan shows decreased perfusion at the left femoral head. Which of the following is the most likely diagnosis? Legg-Calvé-Perthes disease Osteomyelitis Slipped epiphysis Transient synovitis

Correct Answer ( A ) Explanation: The boy's complaints and ancillary studies are consistent with Legg-Calvé-Perthes (LCP) disease. There is osteonecrosis and femoral head deformity due to the temporary interruption of the blood supply to the proximal femoral epiphysis. The peak incidence is from ages four to eight years old. It commonly presents as a limp and activity-related pain that may be localized to the groin or referred to knee or thigh. The limp is described as an antalgic gait where there is shortening of gait phase on the injured side to alleviate weight-bearing pain. On examination, the hip motion is limited, especially internal rotation and abduction. Radiographs (antero-posterior and frog-leg views) are primarily used to diagnose, stage, and prognosticate LCP. Early changes include decreased size of the ossification center, lateralization of the femoral head with widening of the medial joint space, subchondral fracture, and physeal irregularity. In the fragmentation stage, the epiphysis looks fragmented. In the reossification stage, there is new bone formation. In the residual stage, there is gradual remodeling. If the changes are not evident on radiographs, radionuclide bone scanning with technetium-99m is helpful because it can reveal the avascularity of the femoral epiphysis. The features of osteomyelitis (B) include fever, localized pain, and decreased mobility. Slipped epiphysis (C) usually presents in an obese child in early adolescence. There is acute hip pain and inability to walk that occurs after a minor trauma. Transient synovitis (D) is characterized by pain and limitation of motion in the hip following a mild viral infection.

A previously healthy 63-year-old woman presents to your office with a complaint of a painful rash that started two days ago. Physical exam shows a grouping of vesicles on an erythematous base just distal to the right scapula. There are no other skin lesions present. Which of the following is the most appropriate therapy? Acyclovir Amitriptyline Cephalexin Ganciclovir

Correct Answer ( A ) Explanation: Varicella-zoster virus (VZV) is the virus responsible for chickenpox and herpes zoster, also called shingles. Chicken pox is the initial infection that occurs after exposure to the virus. Herpes zoster occurs when the dormant virus is reactivated. Herpes zoster can occur at any age, but is more common in individuals older than 50 years. Clinical presentation of herpes zoster is a painful, unilateral, vesicular rash that occurs in a dermatomal distribution. In patients who are immunocompetent, the vesicles crust in 7-10 days and are no longer considered to be contagious at that point. Diagnosis is determined based on clinical presentation. Treatment decisions are based on duration of symptoms, clinical presentation, patient's age and immune state. Immunocompetent patients older than age 50 years who present within 72 hours of the onset of symptoms should be treated with an antiviral medication such as acyclovir (or valacyclovir) to help decrease symptoms and duration of the infection. The herpes zoster vaccine is recommended for individuals aged 60 years and older to help prevent herpes zoster infection. Amitriptyline (B) has been considered in the treatment of post-herpetic neuralgia. Due to increased side effects in the elderly, it is not recommended in the treatment of varicella zoster virus infections. Cephalexin (C) is used in the treatment of bacterial cellulitis. Ganciclovir (D) is an antiviral medication used to treat cytomegalovirus infections, not VZV infections.

A 45-year-old woman with a history of ethanol abuse presents to the ED complaining of nausea and vomiting. Vital signs are BP 110/70, HR 98, RR 16, and T 38°C. On exam, you note scleral icterus and elicit mild RUQ tenderness of her abdomen. Lab results reveal an AST of 300, ALT of 175, total bilirubin of 2.7, and alkaline phosphatase of 200. A RUQ ultrasound is performed as seen above. Which of the following is the most likely diagnosis? Acute viral hepatitis Alcoholic hepatitis Ascending cholangitis Cholecystitis

Correct Answer ( B ) Explanation: Alcoholic hepatitis is a disease that ranges in clinical severity from subclinical to acute liver failure. Patients often present with nausea, vomiting, anorexia, and abdominal pain. The patient is often tachycardic with a low-grade fever. On exam, you may note scleral icterus or jaundice and the patient often has RUQ tenderness due to inflammation of the liver. There may be characteristic signs of cirrhosis, including spider angiomata, palmar erythema, and gynecomastia. The key finding in differentiating alcoholic hepatitis from viral hepatitis is the elevation in transaminases. In alcoholic hepatitis, there is moderate elevation of AST and ALT, usually only 2-10 times normal. Values in excess of 10 times normal are unusual even in severe cases, unlike in viral or toxin-induced hepatitis than can see a rise into the thousands. Also, the AST is commonly more elevated than the ALT (AST>>ALT). Bilirubin can also be elevated leading to icterus and jaundice. A leukocytosis is often present along with elevation of the PT and INR. The ultraound is consistent with hepatic steatosis, an indication of chronic ethanol abuse. Acute viral hepatitis (A) is defined by significant elevation of transaminases in the acute phase of the infection. AST and ALT are often elevated into the thousands. Ascending cholangitis (C) is most often a consequence of common bile duct blockage by a gallstone, but it can also be caused by blockage from malignancy or a stricture. Patients are usually ill appearing and septic. Lab tests usually reveal elevation of transaminases, alkaline phosphatase, and bilirubin. Ultrasound may demonstrate intrahepatic ductal dilation or stones in the gallbladder suggesting the underlying cause of obstruction. Cholecystitis (D) is usually due to obstruction of the cystic duct by a gallstone. Gallstones are identified in 95% of patients with cholecystitis. Other causes of obstruction include tumor, lymphadenopathy, fibrosis, parasites, and kinking. Physical exam usually reveals RUQ or epigastric tenderness. Lab findings include leukocytosis and normal to mild elevation in transaminases, bilirubin, and alkaline phosphatase. Ultrasound is useful in identifying gallbladder wall thickening, cholelithiasis, and pericholecystic fluid.

A two-year-old child is seen in the emergency room with recent onset of cough and progressive stridor. Respiratory cultures are taken, and you are concerned for bacterial tracheitis. What is the most common organism isolated on respiratory cultures in bacterial tracheitis? Haemophilus influenzae type B Staphylococcus aureus Streptococcus pneumoniae Streptococcus pyogenes

Correct Answer ( B ) Explanation: Bacterial tracheitis can be seen in healthy children between three months and six years of age with a peak occurrence between three and five years old. Parents will often describe URI symptoms prior to bringing their child in to be evaluated. Blood cultures rarely are positive, but respiratory cultures are commonly polymicrobial with staphylococcus aureus the most common isolated species. Bacterial tracheitis is usually diagnosed on a clinical basis. The classic presentation is a child with suspected croup who continues to worsen despite treatment. High fever develops. These patients are at increased risk for airway compromise. They should be immediately started on broad-spectrum intravenous antibiotics. Haemophilus influenzae type B (A), Streptococcus pneumoniae (C), and Streptococcus pyogenes (D) can be isolated on respiratory cultures, however, Staphylococcus aureus is most commonly seen in bacterial tracheitis

A 55-year-old man with a history of peptic ulcer disease presents to your office with a complaint of right knee pain that started last night. On physical exam his knee is erythematous, warm and exquisitely tender to palpation. Synovial fluid analysis reveals the presence of positively birefringent calcium pyrophosphate dihydrate crystals. Which of the following is the most appropriate therapy? Allopurinol Colchicine Indomethacin Vancomycin

Correct Answer ( B ) Explanation: Calcium pyrophosphate crystal deposition disease, also referred to as pseudogout, is a common crystal-induced arthropathy that generally affects the large joints. Pseudogout has a similar clinical presentation to gout, but the etiology is different. Pseudogout may be idiopathic, especially in the elderly. It may also be caused by trauma, hyperparathyroidism, hemochromatosis, and medications that cause hypomagnesemia such as loop diuretics or proton pump inhibitors used in peptic ulcer disease. Patients present with acute onset of severe pain, inflammation and edema in the knees, ankles, elbows or wrists. Pseudogout is generally monoarticular, but may present in multiple joints as well. Diagnosis is by synovial fluid analysis, so arthrocentesis is necessary for patients with monoarticular arthritis. Initial treatment for pseudogout is with nonsteroidal anti-inflammatory drugs (NSAIDs) or colchicine. Workup of the underlying metabolic problem causing the pseudogout attack will help to prevent future flare-ups. Allopurinol (A) is used in the long-term treatment of gout to prevent flares. It should not be used for an acute case of gout. Indomethacin (C) is an NSAID that is first-line in the treatment of acute attacks of both gout and pseudogout. NSAIDs are contraindicated in patients with peptic ulcer disease, heart failure, renal insufficiency, and hypersensitivity to NSAIDs. Vancomycin (D) is used in the treatment of septic arthritis, which can have a similar clinical presentation to both gout and pseudogout.

A 78-year-old woman complains of a progressive burning sensation in both feet. Her past medical history includes hypertension, diabetes type 2, and severe chronic obstructive pulmonary disease. Her examination is significant for ankle weakness, diminished ankle reflexes and intrinsic foot muscle atrophy. Which of the following is the most likely diagnosis? Central cord syndrome Diabetic polyneuropathy Hypoxic brain injury Multiple sclerosis

Correct Answer ( B ) Explanation: Polyneuropathy is a specific term, referring to a generalized, homogenous process affecting many peripheral nerves, while peripheral neuropathy is a less specific term, referring to any peripheral nervous system disorder. Polyneuropathies typically affect distal more than proximal nerves, in a more symmetric than asymmetric fashion. There are several etiologic classes of polyneuropathy, generally differentiated by a few characteristics, which include: demyelinating versus axonal injury, symmetric versus asymmetric, distal versus proximal, chronic versus acute and sensory versus motor. Etiologies include several infectious, inflammatory, toxic and hereditary conditions. The most common polyneuropathies are of the chronic, axonal injury type, with diabetes mellitus and uremia being the most common causes. Other common causes are alcohol abuse and HIV infection. Sensory symptoms usually preceded motor symptoms. Leg symptoms typically precede arm symptoms, in a distal to proximal fashion, giving rise to the classic "stocking-glove" distribution of symptoms. Since the disease process is in the peripheral and not central nervous system, polyneuropathy also displays the motor symptoms of hyporeflexia, flaccidity and atrophy. These "lower-motor-neuron" symptoms are contrasted to the "upper motor neuron" symptoms of hyperreflexia and spasticity, which are exclusive to central nervous system (brain and spinal cord) pathology. (A, C and D) are examples of central nervous system pathology, which do not present with hyporeflexia and atrophy.

A 58-year-old woman presents with progressive orthopnea and peripheral edema. She also gets "winded" when she climbs a full flight of stairs. Which of the following laboratory tests helps define a cardiac versus a pulmonary cause of dyspnea? Beta-2 microglobulin Beta-type natriuretic peptide Erythrocyte sedimentation rate Homovanillic acid

Correct Answer ( B ) Explanation: Cardiomyopathy is defined as a group of diseases which involve the muscle or electrical system of the heart. There are several causes, most of which are genetic in nature. Other etiologies are related to infectious, autoimmune, inflammatory, infiltrative, toxic, electrolytic, endocrine, nutritional and radiation etiologies. There are four main types: dilated, hypertrophic, restrictive and arrhythmogenic-right-ventricular (fibro fatty infiltration of the right ventricle). Dilated cardiomyopathy (DCM) is the most common subtype. It is the third most common cause of cardiac failure, behind coronary artery disease and hypertension. Adult DCM is most commonly caused by hypertension and coronary artery disease, but also is caused by genetic and infectious etiologies. Patients usually present with symptoms of heart failure, such as peripheral and pulmonary edema, cough, orthopnea and dyspnea at rest, with exertion or of the paroxysmal-nocturnal type. Initial evaluation of a patient with these symptoms includes electrocardiography, echocardiography, chest radiography and baseline chemistries, namely Beta-type (Brain) natriuretic peptide (BNP). BNP is secreted by the cardiac myocytes in response to increased volume and filling pressures. Rapid Review Brain Natriuretic Peptide (BNP) ↑ Ventricular myocyte stretch → release ↓ In obese BNP < 100 pg/mL: heart failure unlikely Level does not correlate with heart failure severity Beta-2 microglobulin (A) is used to evaluate hematologic disorders like multiple myeloma, lymphoma and leukemia. It is also associated with multiple sclerosis and other CNS disorders, as well as renal tubular disorders. Erythrocyte sedimentation rate (C) is a marker of inflammation. It is not a reliable test in differentiating cardiac from pulmonary dysfunction. Homovanillic acid (D) is a biomarker of metabolic stress in the central nervous system, not the cardiac or pulmonary systems.

A 4-year-old boy presents with a laceration to his distal forearm. You are considering using a topical tissue adhesive to close the wound. Which of the following is true of tissue adhesive use in lacerations? In wounds that require deep suture closure, the superficial layer should not be closed with skin adhesive Tissue adhesives have the equivalent strength of 4-0 nylon sutures Tissue adhesives should be avoided on areas of cosmetic importance Tissue adhesives should be used only for lacerations <2 cm in length

Correct Answer ( B ) Explanation: Cyanoacrylate tissue adhesives are liquid monomers that polymerize into a stable bond when they come into contact with moisture. They offer many advantages over traditional wound closure with sutures. Their use offers faster wound closure with decreased pain with comparable rates of infection. There is some evidence that indicates that adhesives not only provide their own dressing but also have antimicrobial properties and may decrease the rate of wound infection. Tissue adhesives are equivalent in strength to 4-0 nylon sutures; they should be applied only to areas with appropriate tension. Some limitations include the inability to use antimicrobial or other petroleum-based products on the wound, the recommendation not to swim to limit forces that may prematurely remove the adhesive, and the greater risk of dehiscence. Studies have shown the cosmetic result from tissue adhesive appears (C) to be equivalent to that of standard closure with sutures. Depth of the wound (A) is not a contraindication to tissue adhesive use. Tissue adhesive can be used safely in conjunction with deep absorbable sutures. There is no absolute limit in length (D) of laceration that can be closed with tissue adhesives. The closure method should be guided by factors such as wound tension, exposure to friction and moisture, and ability to approximate wound edges. Tissue adhesive may be applied in high-tension areas or lacerations >4 cm if used in conjunction with subcutaneous or subcuticular sutures.

The leading type of death from an unintentional injury in adults aged 65 and older is due to which of the following? Automobile accidents Falls Fire-related injuries Gunshot wounds

Correct Answer ( B ) Explanation: Falls are the leading type of injury for adults aged 65 and older. Between 30-40% of community-dwelling adults in this age group have at least 1 fall per year. Among older adults, falls are the leading cause of both fatal and nonfatal injuries. Falls are the most common type of unintentional injury in childhood, however the most common cause of death from unintentional injuries in all children and young adults are motor vehicle accidents (A). The proper use of child restraints is the most effective strategy to prevent injury or death. For infants younger than one year, suffocation is the leading cause of death from unintentional injury. Homicide, due to gunshot wounds (D), is the second leading cause of death and suicide is the third leading cause of death in the adolescent population. For older adults, burns and fire-related injuries (C) are the second leading cause of death from accidental injury in the home, however in children and adolescents, fire-related injuries are less common, however are still prevalent and safety precautions should be practiced.

Which of the following is the most common cause of bacterial foodborne disease in the United States? Campylobacter Salmonella Shigella Vibrio cholerae

Correct Answer ( B ) Explanation: In the United States, nontyphoidal Salmonellosis is the most common cause of foodborne disease. It is associated with consumption of milk products, poultry and eggs. Transmission can also occur from pets, specifically reptiles. Patients initially present with diarrhea and colitis. Therapy involves replacing fluids and electrolytes. Patients with severe symptoms including high fever, more than 9-10 stools per day or those requiring hospitalization should be treated with a course of antibiotic therapy. Choice of antimicrobial should be made based on antibiotic resistance patterns and can include fluoroquinolones, trimethoprim-sulfamethoxazole, amoxicillin or intravenous third-generation cephalosporins. Campylobacter (A) is the second most common cause of foodborne disease in the United States and is generally acquired from undercooked poultry. Shigella (C) causes colonic or dysenteric diarrhea and is often found in daycare centers or other institutional settings. Clinical cases of Vibrio cholerae (D) are uncommon although it is considered to be endemic on the gulf coast of the United States and can be found in shellfish.

A previously healthy 65-year-old woman presents to your office with a complaint of thickened and discolored toenails. Previous potassium hydroxide examination of her toenail scrapings shows the presence of dermatophytes. Which of the following is the most appropriate therapy? Oral fluconazole Oral terbinafine Topical medicated chest rub Topical terbinafine

Correct Answer ( B ) Explanation: Onychomycosis is a fungal infection that can affect the toenails or fingernails. There are five different subtypes of onychomycosis that are determined based on clinical features. Patients may present with a combination of these subtypes. Risk factors for onychomycosis include older age, diabetes mellitus, tinea pedis, genetic predisposition, and living with individuals who have onychomycosis. Patients initially present with complaints about the appearance of the nail without physical symptoms. With progression of the infection, thickening, discoloration and deformation of the nails occur and can lead to physical discomfort and pain. Diagnosis is with potassium hydroxide (KOH) examination of nail scrapings. Treatment is indicated for patients with diabetes, with a history of cellulitis, those experiencing pain or physical discomfort and when requested due to cosmetic concerns. First-line treatment is with oral medication. Patients should be advised that recurrence is common. Terbinafine has been found to have the highest cure rate and is taken for six weeks when treating fingernails, twelve weeks when treating toenails. Serum aminotransferases should be monitored before starting treatment with terbinafine and during the treatment process due to hepatotoxicity. Oral fluconazole (A) is sometimes used in the treatment of onychomycosis, but is not as effective as terbinafine and is therefore not recommended as first-line treatment. Topical agents are sometimes used as an adjunct to oral treatment. Medicated chest rubs (C) have been used anecdotally and are unlikely to be harmful, however additional studies need to occur before their use can be recommended. Topical terbinafine (D) is not recommended as first-line therapy for onychomycosis.

A 18-year-old African-American man presents to your office with questions about his diagnosis of sickle cell anemia. He has been managing his condition since childhood by treating his symptoms and having occasional blood transfusions. Now he is experiencing more frequent painful episodes and wants to know if there are other treatment options. Which of the following is the most appropriate pharmacologic treatment? Ferrous sulfate Hydroxyurea Oxycodone Prednisone

Correct Answer ( B ) Explanation: Sickle cell anemia is an autosomal recessive genetic disorder that causes a mutated form of hemoglobin S. This mutation leads to chronic vaso-occlusive crises in affected individuals and causes a number of other health problems. It is most commonly found in persons of African ancestry. Individuals may be carriers of the sickle cell trait, meaning that they carry one mutated gene for hemoglobin S and one normal gene. Carriers generally do not experience the painful or vaso-occlusive crises that those with the disease experience and have some resistance to malaria. Generally diagnosed in early childhood, patients with sickle cell disease experience clinical manifestations that include acute and chronic pain, anemia, splenic sequestration, infection and involvement of multiple organ systems. Aspects of treatment include preventive of infection through immunizations, prevention of vaso-occlusive crises with hydroxyurea and management of painful episodes with rest, analgesia and hydration. Hydroxyurea increases fetal hemoglobin (HbF) production and slightly raises the total hemoglobin concentration in the body. Fetal hemoglobin reduces the chance that red blood cells will sickle in a person who has sickle cell disease. So increased production of HbF can reduce the occurrence of sickling-related complications. Treatment and management of the disease is best provided by a sickle cell disease specialist. Ferrous sulfate (A) is used in the treatment of iron deficiency anemia. Oxycodone (C) may be used for pain control in patients with sickle cell anemia experiencing painful episodes, but is not used for prevention of these episodes. Steroids such as prednisone (D) may be used in the treatment of hemolytic anemia, but are not indicated as first-line treatment for sickle cell anemia.

A 52-year-old man with a long history of alcoholism presents with epigastric pain and vomiting. You diagnose him with pancreatitis. Which of the following laboratory values is associated with an increased risk of mortality on admission? ALT 350 mmol/L LDH 400 IU/L Lipase 14,000 U/L WBC 15,000 cells/mm3

Correct Answer ( B ) Explanation: The Ranson's criteria include a number of parameters which may be used in order to calculate a score predictive of mortality from pancreatitis. The criteria actually has two parts, the first score calculated at the time of admission and the second within 48 hours of admission. With a rising score, the risk of mortality increases and many sources advise consideration of admission to the ICU for a score of 3 or more. An LDH value of >350 IU/L on admission is part of the criteria. The only non-laboratory value included on the admission data of Ranson's criteria is age >55 years. The other correct laboratory data include WBC (D) >16,000, Glucose >200 and AST >250. ALT (A) is not part of the initial criteria. It is also important to note that the lipase (C) level is not part of Ranson's criteria.

What is the abnormal heart sound heard in the above audio recording? Holosystolic murmur S3 S4 Systolic ejection murmur https://www.easyauscultation.com/cases?coursecaseorder=4&courseid=22

Correct Answer ( B ) Explanation: The S3 heart sound, also referred to as a protodiastolic gallop or ventricular gallop, is generally associated with acute heart failure. The sound is associated with early diastolic filling and is heard in such pathologic states as volume overload and left ventricular systolic dysfunction. It occurs at the beginning of diastole and produces a rhythm classically compared to the cadence of the word Kentucky (S1 = Ken; S2 = tuck; S3 = y). The S3 may be normal in people under 40 years of age, may sometimes be heard in pregnancy, and in some athletes but should disappear before middle age. S3 is a dull, low-pitched sound best heard with the bell placed over the cardiac apex with the patient lying in the left lateral decubitus position. Holosystolic murmurs (A) start at S1 and extend up to S2. They are usually due to regurgitation such as in mitral regurgitation, tricuspid regurgitation, or ventricular septal defect. S4 (C) occurs just after atrial contraction and immediately before the systolic S1. It produces a rhythm classically compared to the cadence of the word Tennessee (S4 = Tenn; S1 = ess; S2 = ee). S4 is caused by the atria contracting forcefully in an effort to overcome a hypertrophic ventricle. Systolic murmurs (D) are classified as ejection or regurgitant murmurs. Ejection murmurs emanate from the pulmonic or aortic valves or their surrounding structures. There are many causes of systolic ejection murmurs, including valvular aortic stenosis, atrial septal defect, acute mitral regurgitation, and ventricular septal defect.

An 18-year-old man presents with penile discharge after unprotected sex. A urine GC/Chlamydia test is ordered. Which of the following is an appropriate treatment regimen? Ceftriaxone 125 mg IM X 1 and azithromycin 1,000 mg PO X 1 Ceftriaxone 250 mg IM X 1 and azithromycin 1,000 mg PO X 1 Ciprofloxacin 500 mg BID X 7 days and azithromycin 1 000 mg PO X 1 Wait for the test results in order to tailor treatment

Correct Answer ( B ) Explanation: The patient has urethritis, which is likely caused by either Chlamydia trachomatis or Neisseria gonorrhea. Chlamydial infection and gonorrhea, the 2 most common nonulcerative sexually transmitted infections, also can cause vaginal discharge, especially in the setting of mucopurulent cervicitis, and both tend to cause urethral discharge in men. These 2 infections commonly occur concurrently, and clinical manifestations are clinically indistinguishable. Thus they should be simultaneously treated in all patients. Ceftriaxone 250 mg IM will treat the vast majority of N. gonorrhea strains and azithromycin 1,000 mg PO will treat C. trachomatis. Clinicians should not wait (D) for the test results to return before initiating treatment because there is a risk for disease transmission or the patient is lost to follow-up. Ceftriaxone 125 mg IM (A) is an inadequate dose for treatment of N. gonorrhea. Fluoroquinolones (C) are no longer used for the treatment of gonorrhea because of the high level of resistance. Doxycycline 100 mg PO Q12 X 7 days is an alternative to azithromycin.

Which of the following antiarrhythmic medications is contraindicated in the setting of coronary artery or structural heart disease? Amiodarone Dofetilide Dronedarone Flecainide

Correct Answer ( D ) Explanation: Flecainide is contraindicated in the setting of coronary artery or structural heart disease because of the increased risk of polymorphic ventricular tachycardia. Flecainide is a class IC antiarrhythmic and is an effective agent against both ventricular and supraventricular dysrhythmias. It is often used for atrial fibrillation and its use is sometimes referred to as the "pill-in-the-pocket" approach. In this approach, the patient takes the medication with episodes of paroxysmal atrial fibrillation, rather than taking medication on a daily basis. Flecainide is an attractive agent because of the relatively good side effect profile, efficacy, and ease of use. However, its use is limited, particularly its proarrhythmic effects. Flecainide should also not be used in the setting of sinus or AV node dysfunction, bundle branch block, or long QT syndrome. Patients should take a short acting beta blocker or calcium channel blocker 30 minutes before taking the antiarrhythmic agent or use background rate control therapy. Because of concern of a post conversion pause, patients should be monitored the first time flecainide is used. Dofetilide (B) and amiodarone (A) are class III antiarrhythmics. Class III agents are used for atrial and ventricular arrhythmias but can lengthen the QT interval; for this reason, they are often initiated in an inpatient setting over 3 days to monitor for torsades de pointes. Amiodarone can prolong the QT interval but has not been shown to cause torsades de pointes and does not have to be initiated as an inpatient. Dronedarone (C), the newest agent, can reduce the incidence of hospitalization for cardiovascular events or death in patients with atrial fibrillation or flutter. Dronedarone should not be used in patients with NYHA functional class II or III heart failure with recent decompensation or with class IV heart failure.

A 55-year-old man, smoker, presents to the ED with hemoptysis and dyspnea for four weeks. His vital signs are T 37°C, BP 146/76 mm Hg, HR 85 bpm, RR 20 per minute, and oxygen saturation 96% on RA. His lung exam reveals distant breath sounds on the left side. His chest X-ray is shown above. What is the most likely cause of his hemoptysis? Bronchitis Lung cancer Pneumonia Pulmonary embolism

Correct Answer ( B ) Explanation: This patient has a lung mass that has eroded into a pulmonary vessel. The pulmonary blood supply is under low pressure, so bleeding can present as blood-tinged sputum or gross hemoptysis. Massive hemoptysis is caused by hemorrhage from a bronchial vessel in the majority of cases because bronchial vessels are under systemic blood pressure. Some 80%-90% of patients with cancer as the cause of hemoptysis will have abnormalities on the chest X-ray. Those with normal chest x-rays with a high suspicion for an underlying neoplasm should undergo a chest CT scan and have an outpatient fiber-optic bronchoscopy. Although bronchitis (A) is the most common cause of hemoptysis (responsible for 15%-30% of cases), patients present with cough as the dominant symptom and have abnormal lung exams and normal chest x-rays. The cough may be productive of sputum. The diagnosis of pneumonia (C) requires focal findings on physical exam or infiltrates on radiographic imaging and is typically accompanied by a fever. Patients with lung cancer are at increased risk for pulmonary embolism (D). This patient's Wells score is two (one point each for hemoptysis and malignancy), which makes the likelihood of PE 16% in an ED population. Given the lung mass seen on the chest x-ray, lung cancer is more likely than PE

A 7-year-old boy presents with a fever, cough, runny nose, and conjunctivitis for 3 days followed by the above pictured rash that initially appeared on his face. Exam of his oral mucosa reveals pinpoint gray spots. Which of the following is the most likely diagnosis? Erythema infectiosum Measles Rocky Mountain spotted fever Rubella

Correct Answer ( B ) Explanation: This patient has a typical presentation for measles (rubeola). The measles virus is a respiratory tract pathogen that is highly communicable. After exposure, the virus incubates for 10-14 days. The patient then develops cough, coryza and conjunctivitis along with fever. 2-4 days later, the characteristic rash begins on the face and spreads down the trunk then fades in a head-to-toe direction. Additionally, Koplik's spots, pinpoint gray spots with surrounding red inflammation, develop on the buccal mucosa and are pathognomonic for measles. Treatment is supportive. Erythema infectiosum (A) is usually a nonfebrile disease. Rubella (D) is also characterized by a maculopapular rash and fever but does not have Koplik's spots. Rocky Mountain Spotted Fever (C) results from a tick bite and starts on the extremities and moves centrally.

A recently adopted 6-year-old from an Eastern Europe presents to your office with his parents for a first time visit. His past medical history and physical exam is unremarkable. His parents do not know his immunization history, so you order the appropriate vaccine and immunoglobulin titers in addition to standard testing. His titers are all negative and laboratory work-up is otherwise unremarkable. What immunizations should he receive? DTaP, HepA, HepB, HiB, IPV, MMR, PCV13, Varicella DTaP, HepA, HepB, IPV, MMR, PCV13, Varicella DTaP, HepA, HepB, IPV, MMR, Varicella Tdap, HepA, HepB, IPV, MMR, PCV13, Varicella

Correct Answer ( C ) Explanation: According to the recommended immunization schedule from the Center for Disease Control, the following catch-up immunizations are recommended. DTaP should be administered to children under age 7. Hepatitis A (HepA) vaccine is recommended through age 18. Hepatitis B (HepB) vaccine should be administered in a 3-dose series to all unvaccinated persons. Inactivated poliovirus vaccine (IPV) is recommended in a 4-dose series for children under age 18. Measles, mumps, rubella (MMR) vaccine should be administered in a 2-dose series to all previously unvaccinated children through adolescence. Varicella (VAR) vaccine should be administered to all children through age 18 years without evidence of immunity in a 2-dose series. The following vaccines are not recommended in this patient based on his age and absence of risk factors. Haemophilus influenzae type B (HiB) vaccine (A) is not recommended for patients 5 years and older unless they have specific risk factors (e.g. anatomic or functional asplenia, HIV infection, hematopoietic stem cell transplant). Children 7 years of age and older should receive Tdap (D). Pneumococcal vaccine, for children 6 years or older, PCV13 (B) is only recommended if previously unimmunized against pneumococcus and specific risk factors (i.e. chronic heart disease, chronic lung disease, diabetes mellitus, alcoholism, or chronic liver disease). Incompletely vaccinated children ages 24-59 months should receive one dose of PCV13.

A 44-year-old woman presents with generalized weakness in the proximal muscles, fatigue, headache, weight loss, and abdominal pain with occasional constipation. She has a history of nephrolithiasis one month ago. Physical exam is unremarkable. Laboratory analysis is normal except for a serum calcium level of 10.9 (upper limit of normal is 10.4). Which additional laboratory study should you order next? Angitoensin converting enzyme level Ferritin, iron and TIBC panel Parathyroid hormone Uric acid

Correct Answer ( C ) Explanation: Causes of hypercalcemia are generally divided into two types; primary and secondary. Primary causes are due to excessive parathyroid hormone secretion. Secondary causes include disease processes that directly affect bone metabolism and calcium excretion. The most common cause of primary hyperparathyroidism (PHPT) is a solitary parathyroid adenoma, accounting for approximately 80% of cases. Multiple adenomas are found in 2% to 4% of cases. The second most common cause of PHPT (15%) is parathyroid hyperplasia of multiple (usually < 4) parathyroid glands. The etiologies for these include a mix of congenital and familial diseases (MEN-I, MEN-IIA). Less than 1% of PHPT is caused by primary parathyroid malignancy. PHPT is usually uncovered through routine nonspecific screening laboratory tests (up to 85% of individuals with hyperparathyroidism are asymptomatic) during evaluation for nephrolithiasis, or occasionally in a patient with accelerated osteoporosis and pathologic fracture. There is a classic "quadrad" of symptoms associated with hypercalcemia that, although seen in many disease processes, may be helpful in a patient with hypercalcemia, irrespective of etiology. The mnemonic is "stones, bones, groans, and psychiatric overtones." Labs reveal elevated parathyroid hormone, elevated calcium, and low phosphorus. Angiotensin converting enzyme level (A) may be used in the evaluation of sarcoidosis. Ferritin, iron and TIBC (B) is useful in the workup for iron deficiency anemia. Uric acid (D) is a marker of hyperuricemia. It is useful in the workup of gout and nephrolithiasis.

A 2-year-old previously healthy female presents to the ED with complaints of 3 days of low-grade fever and congestion followed by noisy breathing and cough. Upon examination, you note a frequent barking cough, audible stridor at rest, and retractions. The child does not appear to be in any distress. Which of the following is the most appropriate next step in management? The patient has mild croup and should be discharged after single dose of steroid and 5 mg albuterol nebulizer The patient has moderate croup and should receive oral steroids and be discharged after racemic epinephrine nebulizer with next-day follow-up The patient has moderate croup and should receive oral steroids and racemic epinephrine nebulizer and be observed in the ED for 3 hours and discharged if symptoms abate This patient has mild croup and can be discharged after a single dose of oral steroids

Correct Answer ( C ) Explanation: Croup (laryngotracheitis) is the most common cause of infectious acute upper airway obstruction (stridor). The etiology is viral (parainfluenza, influenza, and RSV) with erythema and swelling of the trachea just below the vocal cords. Patients classically present with a barky or seal-like cough. The mean age of affected patients is 18 months. There is a seasonal increase in autumn and early winter. Because the lungs are not directly affected, oxygen saturation can be maintained, even in severe illness. Aerosolized epinephrine (R or L) decreases airway obstruction. It is indicated for children with stridor at rest or marked work of breathing (tachypnea, retractions, accessory muscle use). Maximal effect is seen within 30 minutes, with potential rebound to baseline within 3 hours. Patients without resting stridor after 3 hours can be safely discharged home. A single dose of oral dexamethasone decreases the need for hospitalization and return ED visits. The patient's croup is moderate, not mild (A and D); therefore, she should receive aerosolized epinephrine to treat the airway obstruction. Albuterol should not be used for croup because stimulation of vascular beta-receptors in the airway may cause vasodilation and worsen airway edema. Mild croup is managed with single-dose steroid, and the patient can usually be safely discharged home. It is prudent to observe patients for at least 3 hours if they receive aerosolized epinephrine (B) because symptoms may recur within this period. Some studies have found that rebound occurred in over 30% of cases after the 2nd hour. If symptoms reoccur, another aerosolized epinephrine treatment is indicated and the patient should be admitted.

A 21-year-old nurse presents after a needle stick from a needle that was uncapped and sticking out of the sharps container. Which of the following is most accurate in terms of HIV prophylaxis? Antiretroviral medications are taken for two weeks Post-exposure prophylaxis decreases rates of seroconversion by 25% Post-exposure prophylaxis started within two hours improves transmission prevention The window for initiation of treatment is 96 hours

Correct Answer ( C ) Explanation: HIV prophylaxis is indicated for a needle stick injury where there is any concern about the possibility of transmission of HIV. The risk of acquiring HIV from an infected needle stick is on average 0.3%. Other factors that play into the likelihood of transmission include the stage of illness of the source patient, depth of injury and whether blood was visible on the device. The most effective treatment is when the first dose of medication is administered within two hours of exposure. Exposed persons are started on antiretroviral medications that prevent HIV from replicating to a degree that is capable of causing seroconversion. The CDC recommends a two or three drug regimen based on the risk of exposure. Some states have their own guidelines established about particular medication. Antiretroviral medication is started immediately and continued for 28 days, not for two weeks (A). This is based on animal data demonstrating the effectiveness of treatment for this duration of time. Rates of effectiveness diminish if the entire course of medication is not completed. Post-exposure prophylaxis decreases rates of seroconversion by more than 50% (B). The one case-control series of occupational exposures showed decreased rates of transmission by 81% with the administration of only one drug (zidovudine). The window for initiation of treatment according to the CDC is 72 hours, not 96 hours (D). Animal data suggests that the window is shorter and probably closer to 36 hours.

A 6-year-old boy presents with fever and on physical exam has a strawberry tongue, red cracked lips, bilateral conjunctival injection, and palmar erythema. Which of the following increases suspicion for Kawasaki disease? Centripetally spreading petechial rash Fine "sand paper" maculopapular rash Generalized non-bullous, non-vesicular rash Macular papular rash starting at the head and spreading caudally

Correct Answer ( C ) Explanation: Kawasaki disease is the leading cause of acquired heart disease in children in the USA. The peak incidence is between the ages 18 and 24 months, with the majority of patients <4 years old and boys are affected more frequently than girls. There are 3 phases of Kawasaki disease. The acute phase lasts 1-2 weeks. This phase presents with the classic symptoms of Kawasaki disease (fever, lymphadenopathy, conjunctivitis, oropharyngeal changes, rash, vasculitis). The subacute acute phase occurs during weeks 2-4 and involves the onset of thrombocytosis (platelets up to 500,000-1,000,000), hand or feet desquamation, and resolution of the fever. The convalescent phase generally begins after 2 months where scarring and calcification of affected coronary arteries may occur. The diagnosis of Kawasaki disease is based on the following criteria: The presence of a generalized non-bullous, non-vesicular rash raises the suspicion for Kawasaki disease. Other conditions that can present with a similar rash include streptococcal disease, which can present with tonsillar exudates (Strep. pharyngitis) or a fine maculopapular "sandpaper" rash (B) as in Scarlet fever. However, conjunctivitis and swelling or erythema of the hands and feet are uncommon. Measles presents with similar symptoms (fever, red eyes, erythematous oropharynx, and rash), however it is rare in vaccinated children and it has a characteristic macular papular rash that starts at the head and spreads caudally (D). Rocky Mountain spotted fever is usually characterized by a centripetally spreading rash that eventually becomes petechial (A).

Which of the following is a valid statement regarding a well-documented adverse effect causally related to a vaccine administration? Inactivated influenza vaccine can cause clinical influenza Measles vaccine is associated with autism The oral polio vaccine causes paralytic disease in immune-deficient individuals Thimerasol, a preservative contained in most vaccines, is associated with mental retardation in infants receiving thimerasol-based vaccines

Correct Answer ( C ) Explanation: Oral polio vaccine is a live attenuated viral vaccine that can lead to paralytic polio in patients with humoral immune deficiency. This complication has an incidence of approximately 1 in 500,000 recipients and is largely the rationale for switching from the live (oral) polio vaccine to the killed (injected) polio vaccine. Inactivated influenza vaccine (A) cannot cause influenza disease in a recipient; only the live wild influenza virus can do so. The misimpression results from the fact that the vaccine is less than optimally effective and the unprotected vaccine recipients get temporally related wild influenza viral infections. Many scientifically rigorous studies performed around the world have proven conclusively that there is no causal relationship between measles vaccination and autism (B). Thimerasol (D), ethyl mercury, has been used as a preservative in vaccines made for human infants and children for decades. The concerns regarding toxicity to infants have been disproven, but it is no longer used in vaccines made for routine use in infants.

A 52-year-old woman presents with progressive central vision loss. She also complains of seeing "red spots" and "floaters". Her medical history is significant for chronic, poorly controlled hypertension and diabetes. She denies trauma and other neurologic symptoms. Which of the following is most likely to be seen during a fundoscopic examination? Koplick spots Lewy bodies Microaneurysms Optic disc edema

Correct Answer ( C ) Explanation: Retinopathy can occur as an acute disease, but more commonly, it is a chronic condition. Two of the most common chronic causes in the US are diabetes and hypertension. Other causes include radiation and sunlight exposure, sickle cell disease, retinal vessel occlusion and trauma. Concerning diabetic retinopathy, the underlying pathology is vessel leakage, microaneurysms and hypervascularization. Eventual edema and hemorrhage lead to vision impairment (especially central vision loss), floaters or red spots, and possible blindness. Fundoscopic examination will reveal several findings, the earliest of which is microaneurysm. Dot and blot, flame-shaped and splinter hemorrhages are also common. Cotton-wool spots, tiny infarcts due to arteriolar occlusion, and macular edema are two common findings in later stages of the disease. Treatment includes laser surgery or vitrectomy. Prevention consists of optimal control of serum glucose and cholesterol, as well as blood pressure. Yearly dilated fundoscopic examination is also recommended. Koplick spots (A) are clustered white lesions of the buccal mucosa that occur a few days prior to measles. Lewy bodies (B) are present in brain tissue, not retina, of elders with dementia. Optic disc edema (D) occurs in optic neuritis (papillitis) and papilledema (increased intracranial pressure state). Macular, not optic, disc edema is more common in diabetic retinopathy.

A patient presents with complaints of edema, malaise and sudsy urine. A microscopic urinalysis shows oval fat bodies and a Maltese cross pattern under polarized light. These findings are most consistent with which of the following diagnoses? Glomerular nephritis Nephritic syndrome Nephrotic syndrome Polycystic kidney disease

Correct Answer ( C ) Explanation: Sudsy or foamy urine indicates high levels of protein in the urine. Oval fat bodies are renal tubular cells which have absorbed filtered lipids, indicating high levels of excess lipids in the urine (i.e. lipiduria). These fat bodies produce a Maltese cross pattern under polarized light. The high levels of protein, along with fat in the urine, point to nephrotic syndrome. Lipiduria is highly sensitive for nephrotic syndrome. Nephrotic syndrome is a constellation of edema, hypoalbuminemia and the urinary excretion of greater than 3 g of protein per day due to a glomerular disorder. Treatment includes ACE inhibitors, fluid and sodium restriction, diuretics for swelling and appropriate management of potential secondary causes. Glomerular nephritis (A) consists of inflammatory proteins being deposited in the glomerular membranes as a result of an immunological response, so urinalysis will show hematuria rather than lipiduria. Nephritic syndrome (B) is characterized by hematuria, proteinuria, hypertension, and mild uremia but no lipiduria. The proteinuria may produce foamy urine and hypoalbuminemia, though the proteinuria in nephritic syndrome tends to be less severe than that of nephrotic syndrome. Polycystic kidney disease (D) is the growth of numerous cysts in the kidneys formed from epithelial cells in the renal tubules and collecting system. Polycystic kidney disease shows proteinuria on urinalysis, but also commonly shows hematuria, pyuria and bacteriuria.

A seven-year-old girl has a history of cerebral palsy, static encephalopathy, seizure disorder, and recent aspiration pneumonia. She completed treatment for pneumonia two days ago but now has developed foul-smelling, watery diarrhea and vomiting. What is the treatment of choice? Intravenous clindamycin Intravenous vancomycin Oral metronidazole Reassurance regarding antibiotic-associated diarrhea

Correct Answer ( C ) Explanation: The onset of diarrhea and vomiting during or closely following a course of antibiotics raises concern for Clostridium difficile infection. Aspiration pneumonia is commonly treated with clindamycin or amoxicillin-clavulanate, both of which may alter the intestinal microbiome and predispose to overgrowth of toxigenic C.difficile. However, any antibiotic may predispose children to C.difficile infection. C.difficile colitis is caused by an intestinal overgrowth of toxigenic C.difficile, followed by production of C.difficile toxin and invasion of the intestinal epithelium. It is best diagnosed by stool toxin PCR. The treatment of choice for mild-to-moderate C.difficile infection is metronidazole, which is both effective and well-tolerated. Patients may also require intravenous hydration if oral hydration is not sufficient to replace stool output, which may be copious. Intravenous clindamycin (A) is not used to treat C.difficile infections. Clindamycin is known to predispose to C.difficile infection. Oral vancomycin can be used to treat C.difficile colitis, but intravenous vancomycin (B) is not recommended. Oral vancomycin is required to reach the intraluminal intestinal C.difficile that has not invaded the mucosa. Oral vancomycin was previously reserved for those who fail metronidazole therapy or have severe disease because of its expense, but the recent availability of generic formulations have decreased its cost. Reassurance regarding antibiotic-associated diarrhea (D) is inappropriate. In observational studies, C.difficile-induced diarrhea has been shown to abate with discontinuation of the offending antibiotic up to two-thirds of cases. However, this child has already completed the course of antibiotics and continues to have copious diarrhea and thus requires treatment.

A 57-year-old man presents for several weeks of fatigue, night sweats, and episodic chills. His exam is remarkable for splenomegaly. A complete blood count shows leukocytosis at 175,000 and mild thrombocytosis. The peripheral blood smear shows a left-shifted myeloid series; blasts are less than 5%. Which of the following additional findings is most consistent with your suspected diagnosis? Auer rods Elevated hematocrit Philadelphia chromosome Rouleaux formation

Correct Answer ( C ) Explanation: The patient has chronic myeloid leukemia (CML) which is confirmed by the presence of the Philadelphia chromosome, a reciprocal translocation between the long arms of chromosomes 9 and 22. CML is a myeloproliferative disorder that occurs most often in middle-aged patients, causing fatigue, night sweats, and low-grade fevers. The physical exam may show splenomegaly and sternal tenderness as marrow overexpansion progresses. In addition to presence of the Philadelphia chromosome, laboratory findings include a median white blood count of 150,000 and peripheral blood showing a left-shifted myeloid series. The red blood cell morphology is normal, and platelets may be normal or slightly elevated. The bone marrow is hypercellular with marrow cells showing less than 5% myeloblasts. The initial treatment of choice for chronic-phase CML is imatinib mesylate (Gleevec®) , a tyrosine kinase inhibitor that is able to normalize hematologic abnormalities and suppress the abnormal Philadelphia chromosome gene in 98 percent of cases. Untreated CML is an unstable disease that can progress to an accelerated blast crisis similar to acute leukemia. An allogeneic stem cell transplant may be needed if medical therapies cannot control the disease, or if the patient has accelerated to a blast crisis. Auer rods (A) are eosinophilic needle-like inclusions that are pathognomonic for acute myeloid leukemia (AML), which would present with a much more rapid course of illness than Chronic myeloid leukemia (CML). Elevated hematocrit (B) is characteristic of polycythemia vera, a myeloproliferative disorder that typically causes headaches, fatigue, and dizziness. Patients with CML will have normal red blood cell morphology. A rouleaux formation (D) is an abnormal red blood cell formation that may occur in multiple myeloma or Waldenström macroglobulinemia.

A 45-year-old businessman presents for "shakiness" of his hands for several months. He notices it most when giving a presentation at work, and adds that his voice "quivers" at those times too. No other symptoms are present. He says his father had a similar problem for most of his adult life. Which of the following medications is the best initial selection for this condition? Botulinum toxin A Donepezil Propranolol Rasagiline

Correct Answer ( C ) Explanation: This patient is showing symptoms of benign essential (familial) tremor, for which the best initial treatment option in lifestyle-limiting disease is the beta-blocker propranolol. Essential tremor usually presents with a postural tremor of the hands or head that is often worsened by psychic stress. When laryngeal muscles are involved, the patient's voice may shake as well. Of note, the legs are usually spared in this condition. Though the cause is uncertain, there appears to be an autosomal-dominant inheritance pattern in some cases. Patients may be initially affected at any age. Essential tremor usually becomes more prominent with age, though significant disability is rare. Symptomatic treatment is limited to patients with lifestyle-limiting tremors. In addition to propranolol, other therapies include primidone, alprazolam, topiramate, or gabapentin. Botulinum toxin A (A) has been shown to have some efficacy in treating essential tremor, but is not considered first-line due to its tendency to cause dose-dependent weakness in patients' injected muscles. Donepezil (B) is a cholinesterase inhibitor that is considered first-line therapy for patients with Alzheimer's disease and dementia with Lewy bodies. There is no known role for it in managing essential tremor. Rasagiline (D), a selective monoamise oxidase B inhibitor, is usually used as adjunctive therapy to levodopa in patients with Parkinsonism. It is not an appropriate choice for essential tremor.

60-year-old woman presents to her primary care provider complaining of a red rash that has developed on her cheeks and nose over the past few years. She states that occasionally the affected area becomes redder and feels like it is stinging or burning especially after drinking coffee. What is the preferred pharmacological treatment? 2.5% hydrocortisone Griseofulvin Metronidazole Mycophenolate mofetil

Correct Answer ( C ) Explanation: This patient likely has rosacea given her physical exam findings and symptoms. While there are many potential treatments, topical metronidazole cream is a first line agent. Rosacea is a very common cause of a red appearing face in Caucasians and people of Mediterranean descent. It most often develops between the ages of 30 and 50, although females tend to present earlier than males. While there are many subtypes of rosacea, there are management techniques that are common among the various types. Patients should avoid items that are known triggers. This often includes spicy foods, alcohol, emotional stress, and hot beverages. Numerous topical therapies may also be effective. Several topical metronidazole preparations may be used, and are significantly more effective in the papulopustular subtype of rosacea. There are four main classifications, which include erythematotelangiectatic rosacea, papulopustular rosacea, phymatous rosacea, and ocular rosacea. In addition to metronidazole, they discuss additional treatments for the various subtypes. Azelaic acid, oral tetracyclines, and vascular laser therapy may also be considered for erythematotelangiectatic and papulopustular rosacea. Oral tetracyclines are the initial therapies for the last two subtypes. Mycophenolate mofetil (D) is an immunosuppressant that inhibits purine biosynthesis. It is used by many patients including those with lupus and rheumatoid arthritis. 2.5% hydrocortisone (A) is typically used to treat atopic dermatitis, not rosacea. Griseofulvin (B) is an antifungal agent that may be used for the treatment of tinea capitis.

A 44-year-old man is transferred from a nursing facility for thrombocytopenia. He was transferred 6 days ago for pulmonary embolism and is on low-molecular weight heparin. His platelets have decreased from 352 to 100. There is no active bleeding at this time. Which of the following is the most likely management indicated? Continue current therapy Order heparin-induced platelet aggregation studies and continue therapy while awaiting results Stop low-molecular weight heparin and start fondaparinux Transfuse platelets

Correct Answer ( C ) Explanation: This patient presents with heparin-induced thrombocytopenia (HIT) requiring immediate cessation of heparin. HIT is an immune-mediated side effect of heparin. It occurs in 0.5 - 2.6% of patients receiving unfractionated heparin and is less common in patients receiving low-molecular weight heparins (<1%). It usually occurs within 5 to 7 days of initiation of therapy. Thrombotic complications are serious and can lead to limb loss in 20% and death in 30%. Diagnosis is made by a drop in platelets by 50% after heparin is started. Serotonin release assays and heparin-induced platelet aggregation assays can confirm the diagnosis. Management focuses on stopping the offending agent (unfractionated or low-molecular weight heparin) and starting a non-related anticoagulant if continued therapy is indicated. Typically, direct thrombin inhibitors (lepirudin, argatroban), factor Xa inhibitors (fondaparinux) or heparinoids (danaproid) are indicated for continued treatment. Although asymptomatic thrombocytopenia does not require therapy at a level of 100, HIT should not be managed by continuation of therapy (A) because of the rate of complications. Therapy should not be continued while awaiting testing (B) as a presumptive diagnosis can be made based on the drop in platelets and heparin treatment. Platelet transfusion (D) is contraindicated unless life-threatening bleeding is present.

Which of the following conditions should prompt administration of the meningococcal vaccine prior to the standard 11 years of age? Cystic fibrosis Diabetes mellitus Hypoplastic left heart Sickle cell anemia

Correct Answer ( D ) Explanation: Children with sickle cell anemia (D) who no longer have a functional spleen should receive the meningococcal vaccine. Patients with sickle cell anemia are at high risk of developing functional or anatomical asplenia due to splenic infarction from vascular occlusion by defective red blood cells. Autosplenectomy dramatically increases the risk of serious infections from encapsulated organisms including Neisseria meningitides, Streptococcus pneumoniae and Haemophilus influenzae. For children under 19 months with functional asplenia, a 4-dose infant series of Meningococcal Groups C and Y and Haemophilus b or Quadrivalent meningococcal conjugate should be administered at 2, 4, 6 and 12 months of age. For children aged 19 through 23 months who have not yet been vaccinated against meningococcus, 2 doses of Meningococcal Groups C and Y and Haemophilus b at least 3 months apart should be administered. For children older than 2 years who have not gotten these vaccines should get 2 primary doses of either Quadrivalent meningococcal conjugate or Meningococcal Groups C and Y and Haemophilus b at least 2 months apart. Routine childhood vaccination entails Meningococcal Groups C and Y and Haemophilus b vaccine at 11 or 12 years with a booster at age 16. Meningitis is an infection of the fluid and lining around the brain and spinal cord and can lead to brain damage, hearing loss, learning disabilities, and even death. Cystic fibrosis (A), diabetes mellitus (B) and hypoplastic left heart (C) are not indications for early meningococcal vaccination, however early 23-valent pneumococcal vaccines are indicated in these patients. Complement deficiency is an indication for vacation against meningococcus with similar guidelines as asplenic patients. Other indications to get an early meningococcal vaccine include HIV infection and travel to hyper-endemic or epidemic locations, such as in the African meningitis belt or the Hajj.

Persistent fever and bacteremia are found in a 28-year-old woman with new onset murmur. She does not use intravenous drugs, and does not have any prosthetic heart valves or history of congenital heart disease. However, she has significant dental disease and poor dentition. What is the most common organism responsible for bacterial endocarditis in this patient? Clostridium septicum Escherichia coli Staphylococcus aureus Streptococcus viridans

Correct Answer ( D ) Explanation: Endocarditis is infection of the cardiac endothelium or valves or both. Acute cases are associated with normal valves and virulent bacteria, while subacute cases represent smoldering infections of abnormal valves with less virulent bacteria. Risk factors include valve prosthesis, history of rheumatic heart disease or prior endocarditis, mitral valve prolapse or regurgitation, congenital cyanotic heart disease, intravenous drug use, indwelling venous catheter, diabetes, poor dentition, hemodialysis and intracardiac devices. Symptoms include persistent bacteremia or fever and other constitutional symptoms (night sweats, weight loss, fatigue, anorexia). Complications include valve defects, septic emboli leading to stroke, pulmonary embolus or myocardial infarction and immune complex deposition disease such as glomerulonephritis and arthritis. The most common cause of native valve endocarditis in a non-user of intravenous drugs is Streptococcus viridans, especially in those with dental disease, as this organism is normally present in the oral cavity. Clostridium septicum (A) is a rare cause of bacterial endocarditis. However, one should consider this as a source of endocarditis in patients with colonic malignancies, as bowel wall degeneration and hematogenous spread of this bacteria can occur. Gram negative rods, like Escherichia coli (B), only account for 5-6% of all endocarditis cases. Staphylococcus aureus (C) is the most common cause of native valve endocarditis in an IV drug abuser, and is more common than S.viridians in those without dental disease.

An 8-month-old infant with a ventricular septal defect has significant growth failure. He also suffers from recurrent pneumonia. Which of the following is the best management of this patient? Atenolol Furosemide Indomethacin Surgical repair

Correct Answer ( D ) Explanation: Infants or children with ventricular septal defects (VSDs) may or may not have symptoms based on the size of the defect. Smaller defects do not allow for a significant left-to-right cardiac shunt, therefore, many of these infants are asymptomatic and carry an excellent prognosis. For small defects, medication is usually not necessary. However, moderate and large defects typically require medication management for congestive heart symptoms. Options include diuretics and digoxin. If these patients fail medication management, surgical repair is recommended. Surgery is also recommended for any VSD that is associated with growth failure or recurrent respiratory infections. Beta-blockers, like atenolol (A), are not recommended to treat the congestive heart symptoms commonly associated with moderate to large VSDs. Although furosemide (B) is a typical first-line treatment of VSD-related congestive heart failure, it would not take precedence over surgical correction for an infant with growth failure, let alone recurrent pulmonary infections. Indomethacin (C) is not a recommended treatment of VSD. It is however used in treating a different congenital heart defect called patent ductus arteriosus.

An otherwise healthy 27-year-old man presents with several days of fever, drenching sweats, and shaking chills one week after returning from India. Which of the following is most likely to reveal the diagnosis? Blood cultures Hepatitis panel India ink stain Thick and thin peripheral smear

Correct Answer ( D ) Explanation: Malaria must be considered in any patient with a history of fever and travel to an endemic region. Four species cause disease in humans: Plasmodium falciparum, Plasmodium ovale, Plasmodium vivax, and Plasmodium malariae. P. falciparum is the most virulent form of the disease able to cause severe organ dysfunction and death. The lifecycle of the organism causes irregular or cyclic fevers in patients that is associated with RBC lysis. Other symptoms include headache, nausea, abdominal pain and upper respiratory complaints. The gold standard includes thick and thin smears of the blood viewed under light microscopy to identify the parasite. Blood cultures (A) are useful for the detection of bacteremia and are part of an evaluation of prolonged fevers. Plasmodia will not grow in a blood culture and the diagnosis will be missed unless a smear is made to specifically look for the parasite. A hepatitis panel (B) is a reasonable consideration for patients with possible exposures to the hepatitis viruses. Hepatitis B and C are transmitted through sexual encounters or blood. Hepatitis A should be considered for patients with exposures to undercooked food, particularly shellfish. An India ink stain (C) is performed for the identification of fungal infections (e.g. Cryptococcus in the cerebrospinal fluid).

A 26-year-old woman just vaginally delivered twin, macrosomic newborn boys. This represents her fourth pregnancy, which has been the longest of them all at 43 weeks gestation. Her delivery was difficult and required forceps to complete. Estimated blood loss was 1100 ml. Currently, she is hypotensive, tachycardic and anemic. Which of the following is the most likely diagnosis? Amniotic fluid embolism Incision dehiscence Rh sensitization Uterine atony

Correct Answer ( D ) Explanation: Postpartum hemorrhage, excessive bleeding after delivery, is defined as blood loss >500 ml. It can be fatal to the mother. The most common cause is uterine atony. Normally after delivery, the uterus will contract, compressing the spiral arteries and placental bed thus preventing hemorrhage. Atony occurs when the normal contraction does not occur. Risk factors for atony are numerous and include general anesthesia, macrosomia, twins or prolonged pregnancy, multiparity, hydramnios, amnionitis and prolonged or difficult labor. An atonic uterus feels soft and boggy. Treatment includes uterine massage and intramuscular methylergonovine maleate or oxytocin, but may require surgical management (artery ligation, artery embolization, hysterectomy). Other causes of postpartum hemorrhage include lower genital tract lacerations or hematomas, retained placenta, uterine inversion and maternal coagulopathy. Amniotic fluid embolism (A) is a rare but often fatal obstetric complication in which amniotic fluid enters the maternal circulation, leading to cardiopulmonary collapse and disseminated intravascular coagulopathy. Dehiscence of a cesarean section incision (B) may also lead to hemorrhage in the postpartum state, but this patient delivered vaginally. In Rh sensitization (C), maternal antibodies are made against fetal red blood cells. With subsequent pregnancies, the placental transmission of these antibodies can lead to a devastating fetal, not maternal, anemia.

A patient complains of a change in bowel habits over the past 3 months. A fecal occult-blood test is positive. During a digital rectal examination, you palpate a solid 2cm by 2cm mass in the rectum. Which of the following serum tests would you order in the initial laboratory evaluation of a patient with suspected rectal neoplasia? Alpha fetoprotein (AFP) Cancer antigen 125 (CA-125) Cancer antigen 15-3 (CA-15-3) Carcinoembryonic antigen (CEA)

Correct Answer ( D ) Explanation: Rectal cancer occurs most commonly as an adenocarcinoma. Other types are lymphoma, sarcoma and carcinoid. Symptoms include frank or occult bleeding, change in bowel habits, abdominopelvic pain, malaise, back pain and urinary symptoms. In addition to a thorough physical examination, evaluation usually requires a digital rectal examination and rigid proctoscopy. Lesions are biopsied for microscopic examination. If metastasis is suspected, further imaging is accomplished via ultrasound, CT and MRI modalities. Laboratory evaluation may include complete blood count, liver and kidney function tests, carcinoembryonic antigen (CEA) test and serum chemistries. Carcinoembryonic antigen is a glycoprotein involved in cell adhesion. It is normally produced in gastrointestinal tissue during fetal development, but the production of CEA stops before birth. Therefore, it is usually present only at very low levels in the blood of healthy adults. However, the serum levels are raised in some types of cancer, which means that it can be used as a tumor marker. Alpha fetoprotein (A) is a tumor marker for hepatocellular carcinoma. CA-15-3 (C) is a tumor marker used to monitor certain cancers, especially breast cancer. CA-125 (B) is most frequently used as a biomarker for ovarian cancer.

A 15-year-old boy with severe asthma is seen in the clinic for a follow-up. He has been hospitalized three times during the past three months due to asthma exacerbations. He required prolonged steroid use to control his asthma. You note abnormal findings on examination that make you suspect Cushing's syndrome. Which exam finding is associated with Cushing's syndrome? Distal myopathy Dry skin Hypotension Supraclavicular fat pads

Correct Answer ( D ) Explanation: The patient has received pharmacologic doses of steroids to control his asthma, which makes him at risk for iatrogenic Cushing's syndrome. The signs and symptoms of Cushing's syndrome result from chronic exposure to excess glucocorticoid. There is a large spectrum of manifestations, depending on duration and intensity of excess steroid production. The most common feature of patients with Cushing's syndrome is progressive central obesity that usually involves the face, neck, trunk, and abdomen. The extremities are usually spared and may be wasted. Enlarged fat pads that fill the supraclavicular fossae and obscure the clavicles are one of the most specific signs. The fat pads make the neck appear thick and shortened. Distal myopathy (A) is not seen in Cushing's syndrome, but instead, there is proximal muscle wasting and weakness that is induced by the catabolic effects of excess glucocorticoid on skeletal muscle. Dry skin (B) is not an expected finding in Cushing's syndrome, but instead, expected skin findings are skin atrophy, purple striae and easy bruisability. Hypotension (C) is not found in hypercortisolism, but instead, common cardiovascular findings include hypertension and dyslipidemia.

A 63-year-old man with prostate cancer and bony metastasis presents with nausea, decreased oral intake, constipation, generalized fatigue, and mild confusion. His vitals are T 37.1°C, HR 102, BP 95/57, RR 20, oxygen saturation 96%, and finger stick blood glucose 102. On examination, he has slow mentation, normal pupil size, dry mucous membranes, and decreased bowel sounds. A chemistry panel shows the following: Na+: 133 K+: 3.9 Cl-: 97 HCO3-: 23 BUN: 27 Cr: -0.8 Ca2+: 13.1 Mg2+: 2.1 Which of the following treatments should be initiated immediately? Bisphosphonates Calcitonin Intravenous furosemide Rapid intravenous normal saline

Correct Answer ( D ) Explanation: The patient's multiple symptoms are caused by hypercalcemia secondary to osteolytic bone metastasis from his prostate cancer. Hypercalcemia often presents with nonspecific symptoms, including fatigue, weakness, confusion, hypertension, bradycardia, polyuria, polydipsia, dehydration, nausea, vomiting, constipation, ataxia, and coma. Any symptomatic hypercalcemia or asymptomatic hypercalcemia with a serum level >14 mg/dL should be aggressively treated. Treatment should start with repleting of intravascular volume with isotonic saline; restoring GFR will lead to renal calcium clearance. Furosemide (C) is a loop diuretic that inhibits resorption of calcium but must be preceded by volume expansion because it will exacerbate dehydration and diminished GFR. Bisphosphonates (A) and calcitonin (B) are osteoclast inhibitors and reduce calcium mobilization from bone. These agents take many hours to work and do not have a role in emergent treatment of hypercalcemia.

You suspect appendicitis in a 30-year-old woman with acute abdominal pain, anorexia, fever and vomiting. Interestingly, she has right lower quadrant pain when the left lower quadrant is palpated. Which of the following signs describes this finding? Dunphy sign Markle sign Psoas sign Rovsing sign

Correct Answer ( D ) Explanation: There are several eponym signs associated with acute appendicitis. This can make documentation confusing. Rovsing sign is the elicitation of right lower quadrant pain when the left lower quadrant is palpated. The cause is likely due to peritoneal irritation of the area localized to the inflamed appendix. Dunphy sign (A) is characterized by increased abdominal pain with coughing, which may also reflect local peritoneal irritation. Markle sign (B) is characterized by abdominal pain when a standing-on-toes patient drops to their heels, which may also reflect local peritoneal irritation. Psoas sign (C) is right lower quadrant pain elicited by right hip extension.

A 75-year-old man with metastatic lung cancer presents with lethargy, nausea, and generalized weakness. Serum calcium level is 14.0 mg/dL. Which of the following is the most appropriate initial treatment? Administration of a loop diuretic Avoidance of medications that cause hypercalcemia Emergent hemodialysis Intravenous fluids

Correct Answer ( D ) Explanation: This patient has hypercalcemia of malignancy. Intravenous fluids are the mainstay of treatment. Hypercalcemia causes an osmotic diuresis, so most patients are hypovolemic. Additional treatments include loop diuretics (once volume has been repleted), calcitonin, and intravenous bisphosphonates. Hypercalcemia is the most common electrolyte disorders associated with cancer, and occurs in approximately 25% of cancer patients. Multiple mechanisms are involved, including local bone breakdown from metastatic bone disease and secretion of parathyroid-hormone-related protein (PTHrP) by tumor leading to increased calcium resorption from bone. Loop diuretics (A) are a second-line treatment for hypercalcemia and should only be administered once volume is repleted. Loop diuretics act to lower serum calcium by inhibiting renal calcium reabsorption. Avoidance of medications which cause hypercalcemia (B) is the treatment for an asymptomatic patient with mildly elevated calcium. Common medications that can lead to hypercalcemia include thiazide diuretics, calcitriol, vitamin D, lithium, and estrogen. Emergent hemodialysis (C) is indicated for patients with severe hypercalcemia with calcium > 18 mg/dL, presence of neurologic symptoms, and when it is inadvisable to give fluids, such as in patients with renal failure or decompensated heart failure.

Which of the following is the most appropriate treatment for stage IV uterine prolapse in a 50-year-old woman with no medical problems and no previous surgical history? Abdominal hysterectomy Anterior colporrhaphy Posterior colporrhaphy Vaginal hysterectomy

Correct Answer ( D ) Explanation: Vaginal hysterectomy is associated with better outcomes and fewer complications than laparoscopic or abdominal hysterectomy. These are general recommendations, but often there are many other considerations including: concomitant procedures needed, concern about failure rates, and restoration of bowel, bladder, and sexual function. In some cases a vaginal hysterectomy is not advisable. These circumstances include a previous intraabdominal surgery for an inflammatory process, such as endometriosis or pelvic inflammatory disease. In such cases, an abdominal laparoscopically assisted vaginal or total laparoscopic hysterectomy may be performed. This can be followed by a vaginal anterior and posterior colporrhaphy, if needed. Uterine prolapse involves the uterus and cervix protruding into or through the barrel of the vagina. It is associated with injuries of the endopelvic fascia, including the cardinal and uterosacral ligaments, as well as injury to the neuromuscular unit with relaxation of the pelvic floor muscles, particularly the levator ani muscles. Major symptoms noted by patients with descensus are a feeling of pelvic pressure, heaviness, fullness, or "falling out" in the perineal area. In cases where the cervix and uterus are low in the vaginal canal, the cervix may be seen protruding from the introitus, giving the patient the impression that a tumor is bulging out of her vagina. When stage IV prolapse has occurred, the patient is aware that a mass has actually prolapsed out of the introitus. Vaginal hysterectomy is associated with better outcomes and fewer complications than laparoscopic or abdominal hysterectomy (A). Operative management of an enterocele with an anterior colporrhaphy (B), or a rectocele with a posterior colporrhaphy (C) is often performed at the time of operation for uterine prolapse after the patient has undergone a hysterectomy, but is not the surgical treatment for uterine prolapse.


Kaugnay na mga set ng pag-aaral

Biology 111 (Kemp) Chapter 19 Test Bank Questions

View Set

B Lymphocytes & Humoral Immunity (18)

View Set

Post Civil War Business and Labor (Praxis 5004)

View Set